Tag Archives: Sohoc

PHƯƠNG TRÌNH NGHIỆM NGUYÊN DẠNG LUỸ THỪA

A. MỘT SỐ CHÚ Ý KHI GIẢI PHƯƠNG TRÌNH DẠNG LŨY THỪA
Nhận xét: Để giải phương trình nghiệm nguyên dạng lũy thừa ta chú ý một số phương pháp thường sử dụng

  • Sử dụng đồng dư để xét tính chẵn lẻ, hay modun của nghiệm.
  • Phân tích thành thừa số.
  • Đánh giá bất đẳng thức.

Do sử dụng nhiều đồng dư, do đó ta chú ý một số tính chất về đồng dư sau Tính chất 3.2. Cho $a$ là một số nguyên tùy ý. Khi đó
(a) $a^2 \equiv 0,1(b\mod 3)$;
(b) $a^2 \equiv 0,1(b\mod 4)$
(c) $a^2 \equiv 0,1,4 (b\mod 8)$;
(d) $a^2 \equiv 0,1,4 (b\mod 5)$;
(e) $a^3 \equiv-1,0,1 (b\mod 7)$
(f) $a^3 \equiv-1,0,1(b\mod 9)$.

Tính chất 3.3. Cho $p$ là một số nguyên tố và $a, b, c, n$ là các số nguyên dương. Ta có
(a) $a^n \vdots p \Leftrightarrow a \vdots p$;
(b) Nếu $a b=p^n$ thì $\left\{\begin{array}{l}a=p^k \\\ b=p^{n-k}\end{array} \quad\right.$ với $k \in \mathbb{N}$ thỏa $0 \leq k \leq n$;
(c) Nếu a b=c^n và (a, b)=1 thì $a=s^n \text { và } b=r^n$ với $s, r \in \mathbb{N}$.

B MỘT SỐ VÍ DỤ
Ví dụ 3.29. Tìm các số nguyên $x, y$ thỏa mān $x^3+1=4 y^2$.

Hướng dẫn giải

Giả sử tồn tại các số nguyên $x, y$ thỏa mãn $x^3+1=4 y^2$. Ta có
$$
x^3=4 y^2-1=(2 y-1)(2 y+1) \text {. }
$$

Đặt $d=(2 y-1,2 y+1)$, ta có $d$ lẻ và $\left\{\begin{array}{l}d \mid 2 y-1 \\\ d \mid 2 y+1\end{array}\right.$.
Do đó $d \mid 2$, suy ra $d=1$ (vì $d$ lẻ). Như vậy $2 y-1$ và $2 y+1$ nguyên tố cùng nhau.
Kết hợp với (3.1) ta suy ra $2 y-1=a^3$ và $2 y+1=b^3$ với $a, b \in \mathbb{Z}$.
Dẫn đến $b^3-a^3=2$ hay $(b-a)\left(b^2+b a+a^2\right)=2$. Từ đó ta được $b=1$ và $a=-1$, suy ra $y=0$ và khi đó $x=-1$. Thử lại thỏa.
Vậy $(x, y)=(-1,0)$.

Ví dụ 3.30. Giải phương trình nghiệm nguyên $x^5+2023 x=5^y+2$.

Hướng dẫn giải

Giả sử tồn tại các số nguyên $x, y$ thỏa mãn $x^5+2023 x=5^y+2$.
Vì $5^y+2$ lẻ nên $x$ lẻ, do đó $x^5+2023 x=x\left(x^4+2023\right) \vdots 4$ (vì $x$ lẻ nên $x \equiv 1(\bmod 4)$.
Tuy nhiên $x^5+2023 x=5^y+2 \equiv 1^y+2 \equiv 3(\bmod 4)$ (Vô lí).

Vậy không tồn tại các số nguyên $x, y$ thỏa mãn $x^5+2023 x=5^y+2$.

Ví dụ 3.31. Tìm các số nguyên $x$ và $y$ sao cho $3^x-y^3=1$.

Hướng dẫn giải

Giả sử tồn tại các số nguyên $x$ và $y$ sao cho $3^x-y^3=1$. Nhận xét $x \geq 0$.
Ta có $3^x=y^3-1=(y+1)\left(y^2-y+1\right)$, suy ra $\left\{\begin{array}{l}y+1=3^t \\\ y^2-y+1=3^{x-t}\end{array} \quad(t \in \mathbb{N}, t \leq x)\right.$.
Khi đó $y=3^t-1$ và
$$
\left(3^t-1\right)^2-\left(3^t-1\right)+1=3^{x-t} \Leftrightarrow 3^{2 t}-3^{t+1}+3=3^{x-t} .
$$

  • Nếu $t=0$, từ (3.2) ta được $1=3^x$ hay $x=0$. Ngoài ra $y=3^0-1=2$.

Nếu $t \geq 1$, giả sử $x-t \geq 2$, khi đó $3^{x-t} \vdots 9$. Từ (3.2) ta có $3^{2 t} \vdots 9$ và $3^{t+1} \vdots 9$ (do $t \geq 1$ ), từ đó suy ra $3 \vdots 9$ (Vô lí).
Do đó $x-t \in{0,1}$.

  • Nếu $x-t=0$ thì $y^2-y+1=1 \Leftrightarrow y(y-1)=0 \Leftrightarrow\left[\begin{array}{l}y=0 \ y=1\end{array}\right.$.
    Với $y=0$ ta tìm được $x=0$ và với $y=1$ ta có $3^x=2$ (Vô lí).
  • Nếu $x-t=1$ thì $y^2-y+1=3 \Leftrightarrow y^2-y-2=0 \Rightarrow y=2$.
    Khi đó $3^x=2^3+1=9$, dẫn đến $x=2$.

Vậy $(x, y)=(0,0)$ hoặc $(x, y)=(2,1)$.

Ví dụ 3.32. Tìm các số nguyên dương $x$ và $y$ sao cho
$$
9^x-7^x=2^y .
$$

Hướng dẫn giải

Giả sử tồn tại các số nguyên dương $x, y$ sao cho $9^x-7^x=2^y$.
Nếu $x$ lẻ thì
$$
9^x-7^x \equiv 1^x-(-1)^x \equiv 2(\bmod 8) .
$$

Do đó $2^y \equiv 2(\bmod 8)$, suy ra $y=1$. Khi đó $9^x-7^x=2 \Rightarrow x=1$.
Nếu $x$ chẵn, đặt $x=2 k\left(k \in \mathbb{N}^*\right)$, ta được
$$
2^y=9^{2 k}-7^{2 k}=\left(9^k-7^k\right)\left(9^k+7^k\right) .
$$

Suy ra
$$
\left\{\begin{array}{l}
9^k-7^k=2^t \\
9^k+7^k=2^{y-t}
\end{array}\right.
$$
với $t \in \mathbb{N}^*$ và $t \leq y$.
– Nếu $k$ lẻ, khi đó $2^t \equiv 9^k-7^k \equiv 2(\bmod 8)$, do đó $t=2$ và $k=1$.
Dẫn đến $x=2$ và $2^y=81-49=32 \Rightarrow y=5$.
– Nếu $k$ chẵn, ta có
$$
9^k+7^k \equiv 1^k+(-1)^k \equiv 2(\bmod 8) .
$$

Do đó $2^{y-t} \equiv 2(\bmod 8)$, suy ra $y-t=1$. Như vậy $9^k+7^k=2$ (Vồ lí).
Vậy $(x, y)=(1,1)$ hoặc $(x, y)=(2,5)$.

Ví dụ 3.33. Tìm tất cả các số nguyên tố $p$ sao cho luôn tồn tại các số nguyên dương $n, x, y$ thỏa mãn
$$
p^n=x^3+y^3 .
$$

Hướng dẫn giải

Đặt $x=p^t x_1$ và $y=p^s y_1\left(x_1, y_1, s, t \in \mathbb{N}\right.$ và $\left.x_1, y_1 \neq p\right)$.
Ta có
$$
p^n=p^{3 t} x_1^3+p^{3 s} y_1^3>p^{3 t} \Rightarrow n>3 t .
$$

Không mất tính tổng quát, giả sử $t \geq s$.
Nếu $t>s$ thì $p^{n-3 s}=p^{3(t-s)} x_1^3+y_1^3 \vdots p \Rightarrow y_1^3 \vdots p$ (Vô lí).
Vậy $t=s$, do đó $p^{n-3 t}=x_1^3+y_1^3=\left(x_1+y_1\right)\left(x_1^2-x_1 y_1+y_1^2\right)$.

  • Nếu $x_1^2-x_1 y_1+y_1^2=1$ thì $x_1=y_1=1$.
    Khi đó $p^{n-3 t}=2 \Rightarrow\left\{\begin{array}{l}p=2 \\\ n-3 t=1\end{array} \Rightarrow\left\{\begin{array}{l}p=2 \\\ n=3 t+1\end{array}\right.\right.$.
    Lúc này ta được $x=y=2^t$. Thử lại thỏa.
  • Nếu $x_1^2-x_1 y_1+y_1^2>1$, ta được
    $$
    \left\{\begin{array}{l}
    x_1+y_1=p^k \\\
    x_1^2-x_1 y_1+y_1^2=p^{n-3 t-k}
    \end{array}\right.
    $$
    với $k \geq 1, n-3 t-k \geq 1$.

Do đó $\left(x_1+y_1\right)^2-\left(x_1^2-x_1 y_1+y_1^2\right)=3 x_1 y_1 \vdots p \Rightarrow 3 \vdots p \Rightarrow p=3$.

Ngoài ra, nếu $n-3 t-k \geq 2$ thì $x_1^2-x_1 y_1+y_1^2=\left(x_1+y_1\right)^2-3 x_1 y_1 \vdots 3^2$, mà $\left(x_1+y_1\right)^2 \vdots 3^2$ nên $3 x_1 y_1 \vdots 3^2 \Rightarrow x_1 y_1 \vdots 3$ (Vô lí).
Vậy $n-3 t-k=1$ hay $x_1^2-x_1 y_1+y_1^2=3$. Không mất tính tổng quát, giả sử $x_1 \geq y_1$ thì ta được $x_1=2$ và $y_1=1$.
Từ đây ta được $n-3 t=2 \Leftrightarrow n=3 t+2$ và $x=2 \cdot 3^t$ và $y=3^t$.
Thử lại thỏa.
Vậy $p=2$ và $p=3$ là các số nguyên tố cần tìm.

Ví dụ 3.34. Tìm nghiệm tự nhiên của phương trình
$$
\left(2^x+1\right)\left(2^x+2\right)\left(2^x+3\right)\left(2^x+4\right)-5^y=11879 .
$$

Hướng dẫn giải

Giả sử tồn tại các số tự nhiên $x, y$ thỏa mãn
$$
\left(2^x+1\right)\left(2^x+2\right)\left(2^x+3\right)\left(2^x+4\right)-5^y=11879 .
$$

Ta có
$$
\begin{aligned}
\left(2^x+1\right)\left(2^x+2\right)\left(2^x+3\right)\left(2^x+4\right) & =\left(4^x+5 \cdot 2^x+4\right)\left(4^x+5 \cdot 2^x+6\right) = \left(4^x+5 \cdot 2^x+5\right)^2-1 .
\end{aligned}
$$

Do đó $\left(4^x+5 \cdot 2^x+5\right)^2-1-5^y=11879 \Leftrightarrow\left(4^x+5 \cdot 2^x+5\right)^2-5^y=11880$.
Nếu $y \geq 1$ thì ta suy ra $4^x+5 \cdot 2^x+5 \vdots 5 \Rightarrow 4^x \vdots 5$. (Vô lí)
Do đó $y=0$, khi đó
$$
\left(4^x+5 \cdot 2^x+5\right)^2=11881 \Rightarrow 4^x+5 \cdot 2^x+5=109 \Leftrightarrow 4^x+5 \cdot 2^x-104=0 .
$$

Suy ra $2^x=8 \Rightarrow x=3$.
Vậy $x=3$ và $y=0$.

Ví dụ 3.35. Cho $M=a^2+3 a+1$ với $a$ là số nguyên dương.
(a) Chứng minh rằng mọi ước của $M$ đều là số lẻ.
(b) Tìm các giá trị của $a$ để $M$ là lũy thừa của 5 .

Hướng dẫn giải

(a) Ta có $a^2+3 a+1=a(a+3)+1$ là số lẻ. Do đó mọi ước của $M$ đều là số lẻ.
(b) Giả sử tồn tại $n \in \mathbb{N}^*$ thỏa mãn $a^2+3 a+1=5^n$. Khi đó
$$
a^2+3 a-4=5^n-5 \Leftrightarrow(a+4)(a-1)=5\left(5^{n-1}-1\right) .
$$

Nếu $n>1$ thì $5^{n-1}-1>0$.
Ta lại có $(a+4)(a-1) \vdots 5$ và $a+4-(a-1)=5$ nên $\left\{\begin{array}{l}a+4 \vdots 5 \\\ a-1 \vdots 5\end{array}\right.$.
Do đó $(a+4)(a-1) \vdots 25 \Rightarrow 5\left(5^{n-1}-1\right) \vdots 25 \Rightarrow 5^{n-1}-1 \vdots 5$. (Vô lí)
Vậy $n=1$ hay $a^2+3 a+1=5 \Rightarrow a=1$.
Thử lại thỏa, vậy $M$ là lũy thừa của 5 khi và chỉ khi $a=1$.

Ví dụ 3.36. Tìm số tự nhiên $n$ sao cho $8^n+47$ là số nguyên tố.

Hướng dẫn giải

  • Xét $n=2 k(k \in \mathbb{N})$, khi đó
    $$
    p^n \equiv 8^n+47 \equiv(-1)^{2 k}+47 \equiv 48 \equiv 0(\bmod 3) .
    $$

Do đó $p$ ! 3 nên $p$ không là số nguyên tố (Vô lí).

  • Xét $n=4 k+1\left(k \in \mathbb{N}^*\right)$, khi đó
    $$
    p \equiv\left(8^4\right)^k \cdot 8+47 \equiv 8+47 \equiv 55 \equiv 0(\bmod 5) .
    $$

Do đó $p \vdots: 5$ nên $p$ không là số nguyên tố (Vô lí).

  • Nếu $n=4 k+3\left(k \in \mathbb{N}^*\right)$, khi đó
    $$
    p \equiv\left(8^4\right)^k \cdot 8^3+47 \equiv 8^3+47 \equiv 559 \equiv 0(\bmod 13) .
    $$

Do đó $p$ : 13 nên $p$ không là số nguyên tố (Vô lí).
Vậy không tồn tại số tự nhiên $n$ để $8^n+47$ là số nguyên tố.

Ví dụ 3.37. Cho phương trình $2^x+5^y=k^2$ ( $x, y, k$ là các số nguyên dương).
(a) Chứng minh rằng phương trình trên vô nghiệm khi $y$ chẵn.
(b) Tìm $k$ để phương trình có nghiệm.
(Đề thi tuyển sinh vào lớp 10 chuyên toán PTNK 2022)

Hướng dẫn giải

(a) Giả sử tồn tại $y \in \mathbb{N}^*$ chẵn để phương trình trên có nghiệm.

  • Với $x=1$ thì $2+5^y=k^2 \equiv 2(\bmod 5)$.
    Điều này vô lý vì $k^2 \equiv 0,1,4(\bmod 5)$ với mọi $k \in \mathbb{N}$.
  • Với $x>1$, do $y$ chẵn nên ta đặt $y=2 m(m \in \mathbb{N})$.
    Khi đó
    $$
    2^x+5^{2 m}=k^2 \Leftrightarrow 2^x=\left(k-5^m\right)\left(k+5^m\right) \Rightarrow\left\{\begin{array}{l}
    k-5^m=2^t \\\
    k+5^m=2^{x-t}
    \end{array} \quad(t \geq 0) .\right.
    $$

Vì $k+5^m>k-5^m$ nên $x-t>t$, suy ra $k=2^{t-1}+2^{x-t-1}$.
Ta thấy nếu $t=0$ thì $k=\dfrac{1}{2}+2^{x-1} \notin \mathbb{N}$. Do đó $t \geq 1$.

Mặt khác $k$ lẻ và $t-1<x-t-1$ nên $2^{t-1}=1 \Rightarrow t=1$. Khi đó $k-5^m=2 \Leftrightarrow k=2+5^m$. Thay vào $2^x+5^{2 m}=k^2$, ta được
$$
2^x+5^{2 m}=\left(2+5^m\right)^2 \Leftrightarrow 2^x=4+2 \cdot 5^m .
$$

Vì $x>1$ nên $2^x \vdots 4$, suy ra $2 \cdot 5^m \vdots 4$ (Vô lí).
Vậy phương trình vô nghiệm khi $y$ chẵn.
(b) Giả sử phương trình có nghiệm, khi đó $y$ lẻ.

  • Nếu $x=4 z+1(z \in \mathbb{N})$ thì
    $$
    k^2 \equiv 2^x+5^y \equiv 2^{4 z} \cdot 2+5^y \equiv 2(\bmod 5) .
    $$

Điều này vô lý vì $k^2 \equiv 0,1,4(\bmod 5)$ với mọi $k \in \mathbb{N}$.

  • Nếu $x=4 z+3(z \in \mathbb{N})$ thì
    $$
    k^2 \equiv 2^{4 z} \cdot 2^3+5^y \equiv 8 \equiv 3(\bmod 5) \text { (Vô lí). }
    $$

Vậy $x$ chẵn, đặt $x=2 t\left(t \in \mathbb{N}^*\right)$.
Ta có
$$
2^x+5^y=k^2 \Leftrightarrow 5^y=\left(k-2^t\right)\left(k+2^t\right) \Rightarrow\left\{\begin{array}{l}
k-2^t=5^s \\\
k+2^t=5^{y-s}
\end{array} \quad(s \in \mathbb{N}) .\right.
$$

Nếu $s>0$ thì $5^{y-s}-5^s \vdots 5$ nên $2^{t+1} \vdots 5$ (vô lý). Do đó $s=0$.

Khi đó $\left\{\begin{array}{l}k=1+2^t \\\ k=5^y-2^t\end{array}\right.$. Suy ra $1+2^t=5^y-2^t \Rightarrow 5^y-1=2^{t+1}$.
Nếu $t>1$ thì $2^{t+1} \vdots 8$. Dặt $y=2 l+1$, khi đó
$$
2^{t+1}=5^y-1=25^l \cdot 5-1 \equiv 5-1 \equiv 4(\bmod 8) \text{vô lý}
$$

Vậy $t=1$, suy ra $k=3$. Với $k=3$, ta tìm được $x=2$ và $y=1$.
Vậy phương trình có nghiệm khi và chỉ khi $k=3$.

Ví dụ 3.38. Cho $k$ là số nguyên dương và $a=3 k^2+3 k+1$.
(a) Chứng minh rằng $2 a$ và $a^2$ là tổng của ba số chính phương.
(b) Chứng minh rằng nếu $a$ là uớc của số nguyên $b$ và $b$ bằng tổng của ba số chính phương thì bất kì lũy thừa với số mũ nguyên dương nào của $b$ cũng là tổng của ba số chính phương.

Hướng dẫn giải

(a) Ta có
$$
\begin{aligned}
2 a=6 k^2+6 k+2 & =k^2+\left(k^2+2 k+1\right)+\left(4 k^2+4 k+1\right) = k^2+(k+1)^2+(2 k+1)^2
\end{aligned}
$$
$$
\begin{aligned}
a^2 & =\left(3 k^2+3 k-1+2\right)^2=9 k^4+18 k^3+15 k^2+6 k+1 = \left(4 k^4+12 k^3+13 k^2+6 k+1\right)+\left(4 k^4+4 k^3+k^2\right)+\left(k^4+2 k^3+k^2\right) = \left(2 k^2+3 k+1\right)^2+\left(2 k^2+k\right)^2+\left(k^2+k\right)^2
\end{aligned}
$$
(b) Đặt $a^2=a_1^3+a_2^3+a_3^3$ với $a_1, a_2, a_3 \in \mathbb{Z}$.
Đặt $b=c a$ với $c$ là số nguyên dương, do $b$ bẳng tổng của ba số chính phương nên $b=b_1^2+b_2^2+b_3^2$ với $b_1, b_2, b_3$ là các số nguyên.
Xét số nguyên dương $n$ bất kì, khi đó

  • Nếu $n=2 k\left(k \in \mathbb{Z}^{+}\right)$thì
    $$
    \begin{aligned}
    b^n & =c^{2 k} a^{2 k}=\left(c^k a^{k-1}\right)^2 a^2 = \left(c^k a^{k-1}\right)^2\left(a_1^2+a_2^2+a_3^2\right) = \left(c^k a^{k-1} a_1\right)^2+\left(c^k a^{k-1} a_2\right)^2+\left(c^k a^{k-1} a_3\right)^2
    \end{aligned}
    $$
  • Nếu $n=2 k+1(k \in \mathbb{Z})$ thì
    $$
    b^n=\left(b^k\right)^2 \cdot b=\left(b^k\right)^2\left(b_1^2+b_2^2+b_3^2\right)=\left(b^k b_1\right)^2+\left(b^k b_2\right)^2+\left(b^k b_3\right)^2
    $$

Hoàn tất chứng minh.


C. CÁC BÀI TẬP RÈN LUYỆN

Bài 3.13. Tìm nghiệm nguyên dương của phương trình
$$
x^3+x^2+x+1=2011^y .
$$

Bài 3.14. Tìm tập nghiệm nguyên dương của phương trình
$$
8^x+15^y=17^z .
$$

Bài 3.15. Tìm các số nguyên dương $x, y, z>1$ thỏa mãn
$$
(x+1)^y-x^z=1 .
$$

Bài 3.16. Tìm nghiệm tự nhiên của phương trình $5^x-3^y=2$.

Bài 3.17. Tìm nghiệm nguyên dương của phương trình
$$
2^x \cdot 3^y+5^z=7^t .
$$

Bài 3.18. Cho các số nguyên dương $m, n \geq 2$. Tìm nghiệm nguyên dương của phương trình
$$
x^n+y^n=3^m .
$$

Bài 3.19. Cho $p$ là một số nguyên tố và $a, n$ là các số nguyên dương. Chứng minh rằng nếu $2^p+3^p=$ $a^n$ thì $n=1$.

Bài 3.20. Chứng minh rằng tích của ba số nguyên liên tiếp không thể là lũy thừa với số mũ lớn hơn 1 của một số nguyên.

Bài 3.21. Cho phương trình $3 x^2-y^2=23^n$ với $n$ là số tự nhiên.
(a) Chứng minh nếu $n$ chẵn thì phương trình đã cho không có nghiệm nguyên $(x, y)$.
(b) Chứng minh nếu $n$ lẻ thì phương trình đã cho có nghiệm nguyên $(x, y)$.

Bài 3.22.
(a) Cho $m$ là số nguyên. Chứng minh rằng nếu tồn tại các số nguyên $a, b, c$ khác 0 sao cho $a+b+c=0$ và $a b+b c+c a+4 m=0$ thì cũng tồn tại các số nguyên $a^{\prime}, b^{\prime}, c^{\prime}$ sao cho $a^{\prime}+b^{\prime}+c^{\prime}=0$ và $a^{\prime} b^{\prime}+b^{\prime} c^{\prime}+a^{\prime} c^{\prime}+m=0$.
(b) Với $k$ là số nguyên dương, chứng minh rằng không tồn tại các số nguyên $a, b, c$ khác 0 sao cho $a+b+c=0$ và $a b+b c+c a+2^k=0$.
(Đề thi tuyển sinh lớp 10 chuyên Toán PTNK 2015)

ƯỚC CHUNG VÀ MỘT SỐ ÁP DỤNG

Bài viết của thầy Nguyễn Vĩnh Khang – Giáo viên Star Education

Các tính chất của ước chung

Nhận xét: Nếu ta đặt $(x, y)=d$, thì $x^{\prime}=\dfrac{x}{d}$ và $y^{\prime}=\dfrac{y}{d}$ nguyên tố cùng nhau. Từ đó lợi dụng các tính chất liên quan đến số nguyên tố cùng nhau như (được sử dụng thẳng, không cần chứng minh)

  • Nếu $a b: c$, và $(b, c)=1$, ta có $a: c$.
  • Nếu $a: b$ và $a: c$, với $(b, c)=1$, ta có $a: b c$.
  • Nếu $(a, b)=1, r$ là ước của $a, s$ là ước của $b$, ta cũng có $(r, s)=1$.
    để phân tích bài toán tiếp. Việc đặt ước chung như vậy sẽ làm đơn giản bài toán (do ta có thể rút $d$ ra rồi triệt tiêu, nếu được) và cho thêm dữ kiện $\left(x^{\prime}, y^{\prime}\right)=1$.

Tính chất 3.1. Giả sử $a, b, c, n$ là các số nguyên dương, chứng minh những tính chất sau
(a) $\operatorname{gcd}(a, b, c)=\operatorname{gcd}(\operatorname{gcd}(a, b), c)$
(b) $\operatorname{gcd}(a c, b c)=\operatorname{gcd}(a, b) c$
(c) Nếu $\operatorname{gcd}(a, b)=1$, ta có $\operatorname{gcd}(a b, c)=\operatorname{gcd}(a, c) \operatorname{gcd}(b, c)$
(d) $\operatorname{gcd}\left(a^n, b^n\right)=\operatorname{gcd}(a, b)^n$.

Chứng minh.
Phần 1: gọi $d=\operatorname{gcd}(a, b, c)$ ta có $d$ là ước của $a, b$, nên $\operatorname{gcd}(a, b)$ : $d$. Nhưng $c: d$, nên ta được một chiều
$$
\operatorname{gcd}(\operatorname{gcd}(a, b), c) \vdots d=\operatorname{gcd}(a, b, c)
$$

Để chứng minh chiều còn lại, gọi $d=\operatorname{gcd}(\operatorname{gcd}(a, b), c)$. Tương tự như trên ta có $d$ là ước của $\operatorname{gcd}(a, b)$, nên $d$ cũng là ước của $a, b$. Nhưng $d$ là ước của $a, b, c$, nên
$$
\operatorname{gcd}(a, b, c) \vdots d=\operatorname{gcd}(\operatorname{gcd}(a, b), c)
$$

Kết hợp (1.1) và (1.2), ta có đpcm.

Phần 2: nếu $d=(a c, b c)$, ta có $d: c$ do $c$ là ước chung của $a c, b c$. Đặt $d=k c$, ta có $(a c, b c)=k c$, và $a c, b c: k c$. Nói cách khác $a, b: k$, nên $(a, b): k$, và
$$
c(a, b) \vdots k c=(a c, b c)
$$

Mặt khác, đặt $k=(a, b)$, ta có $a, b: k$, nên $a c, b c: k c$. Theo định nghīa, $(a c, b c) \vdots k c=(a, b) c$. Kết hợp với (2.1) ta có đpem $\operatorname{gcd}(a c, b c)=\operatorname{gcd}(a, b) c$.

Phần 3: gọi $k=\operatorname{gcd}(a, c), l=\operatorname{gcd}(b, c)$, theo tính chất 2 , ta được
$$
\left\{\begin{array}{l}
\operatorname{gcd}\left(\dfrac{a}{k}, \dfrac{c}{k}\right)=1 \\
\operatorname{gcd}\left(\dfrac{b}{l}, \dfrac{c}{l}\right)=1
\end{array}\right.
$$

Mặt khác $a: k, b: l$, nhưng $a, b$ lại nguyên tố cùng nhau, nên $k, l$ cūng vậy. Kết hợp với $c: k, l$, ta có $c: k, l$. Để ý rằng $\dfrac{c}{k l}$ là ước của $\dfrac{c}{k}$ và $\dfrac{c}{l}$, nên
$$
\left\{\begin{array}{l}
\operatorname{gcd}\left(\dfrac{a}{k}, \dfrac{c}{k l}\right)=1 \\
\operatorname{gcd}\left(\dfrac{b}{l}, \dfrac{c}{k l}\right)=1
\end{array}\right.
$$

Ta chứng minh $\operatorname{gcd}(a b, c)=1$ nếu $\operatorname{gcd}(a, b)=\operatorname{gcd}(b, c)=\operatorname{gcd}(a, c)=1$. Thật vậy, giả sử ngược lại, tức $\operatorname{gcd}(a b, c) \neq 1$. Khi đó tồn tại $p$ là ước nguyên tố chung của $a b, c$. Nhưng $a b: p$ thì ta phải có $a: p$ hoặc $b: p$, nên $\operatorname{gcd}(a, c): p$ hoặc $\operatorname{gcd}(b, c)$ : $($ cả 2 đều mâu thuẫn với giả thiết).

Áp dụng quan sát trên cho (3.1), ta được
$$
\operatorname{gcd}\left(\dfrac{a b}{k k}, \dfrac{c}{k l}\right)=1 \Leftrightarrow \operatorname{gcd}(a b, c)=k l=\operatorname{gcd}(a, c) \operatorname{gcd}(b, c)
$$

Phần 4: ta chứng minh $\operatorname{gcd}\left(a^n, b^n\right)=1$ nếu $\operatorname{gcd}(a, b)=1$. Thật vậy, giả sử $\operatorname{gcd}\left(a^n, b^n\right) \neq 1$, khi đó $a^n, b^n$ phải có một ước nguyên tố chung $p$. Sử dụng tính chất nếu $x y: p$ thì $x: p$ hoặc $y: p$. Từ đó $a, b: p$, vô lý.

Đặt $d=\operatorname{gcd}(a, b)$, ta có $\operatorname{gcd}\left(\dfrac{a}{d}, \dfrac{b}{d}\right)=1$, nên
$$
\operatorname{gcd}\left(\left(\dfrac{a}{d}\right)^n,\left(\dfrac{b}{d}\right)^n\right)=1
$$

Nhân $d^n$ cho cả 2 vế, và dùng tính chất 2 , ta được
$$
\operatorname{gcd}(a, b)^n=d^n=d^n \operatorname{gcd}\left(\left(\dfrac{a}{d}\right)^n,\left(\dfrac{b}{d}\right)^n\right)=\operatorname{gcd}\left(a^n, b^n\right)
$$

Hệ quả 3.1
Giả sử $a, b, c, n$ là các số nguyên dương, chứng minh những tính chất sau
(a) Nếu $a b: c$ và $(a, b)=1$, tồn tại $k, l$ sao cho $k l=c$, và $a: k, b \vdots l$.
(b) Nếu $a b=c^n$ và $(a, b)=1(n \geq 2)$, tồn tại $k, l$ sao cho $k l=c$ và $a=k^n, b=l^n$.

Chứng minh.

Phần 1: gọi $k=\operatorname{gcd}(a, c), l=\operatorname{gcd}(b, c)$, theo bài tập trước, ta có $k l=\operatorname{gcd}(a, c) \operatorname{gcd}(b, c)=$ $\operatorname{gcd}(a b, c)=c$, và $a: k, b: l$ theo định nghĩa.

Phần 2: gọi $k=\operatorname{gcd}(a, c), l=\operatorname{gcd}(b, c)$, theo bài tập trước, ta có $k l=\operatorname{gcd}(a, c) \operatorname{gcd}(b, c)=\operatorname{gcd}(a b, c)=$ c. Mặt khác
$$
k^n=\operatorname{gcd}\left(a^n, c^n\right)=\operatorname{gcd}\left(a^n, a b\right)=a \operatorname{gcd}\left(a^{n-1}, b\right)=a
$$
, ở đây $\operatorname{gcd}\left(a^{n-1}, b\right)=1$ do nếu tồn tại $p$ là ước nguyên tố chung cho $a^{n-1}, b$, ta phải có $p$ là ước chung của $a, b$ (vô lý). Chứng minh tương tự, ta cũng có $l^n=b$. Ta có đpcm.

B. MỘT SỐ VÍ DỤ ÁP DỤNG
Ví dụ 3.1 (Junior Balkan Mathematical Olympiad 2001).
Tìm ước chung lớn nhất của $A_0, A_1, A_2, \ldots, A_{1999}$, với $A_n=2^{3 n}+3^{6 n+2}+5^{6 n+2}$.

Hướng dẫn giải

Do $A_0=35=5 \cdot 7$, nên ước chung lớn nhất, gọi là $d$, phải là 1 trong 4 số ${1,5,7,35}$. Do $A_1=$ $2^3+3^8+5^8 \equiv 8+(-2)^8 \equiv 4(\bmod 5)$ nên $d \neq 5,35$. Mặt khác, theo định lý Fermat, ta có $3^6 \equiv 5^6$ $(\bmod 7)$, nên
$$
A_n \equiv 8^n+\left(3^6\right)^n \cdot 9+\left(5^6\right)^n \cdot 25 \equiv 1+9+25 \equiv 0 \quad(\bmod 7)
$$
Ta kết luận $d=7$.


Ví dụ 3.2. Chứng minh rằng nếu $d>0$ không phải là số chính phương, thì $\sqrt{d}$ là số vô tỷ.

Hướng dẫn giải

Để ý rằng $d=1^2 \cdot d$ nên $d$ luôn có thể viết thành dạng $d=x^2 y$ (với $x, y>0$ ). Chọn $x$ lớn nhất có thể, và để ý $y \neq 1$. Nếu $y$ có ước chính phương $z^2$ ngoài 1 , thì $d=x^{\prime 2} y^{\prime}$, với $x^{\prime}=x z>x$ và $y^{\prime}=\dfrac{y}{z}$, vô lý. Như vậy $y$ là tích các số nguyên tố khác nhau (do nếu $p$ là ước nguyên tố của $y$, thì $\dfrac{y}{p}$ không thể nào chia hết cho $p$ được).

Giả sử $\sqrt{d}=\dfrac{a}{b}$ là một số hữu tỷ, với $a, b$ nguyên dương nguyên tố cùng nhau. Ta có $$ a^2=b^2 d=(b x)^2 \cdot y $$ nên $a^2: y$. Nhưng $y$ chỉ là tích các số nguyên tố khác nhau, nên $a: y$. Thế $a=c y$ vào (*), ta được
$$
c^2 y^2=(b x)^2 y \Leftrightarrow b^2 x^2=c^2 y
$$

Để ý $c^2 y: b^2$, nhưng $(c, b)=1$ (do $(a, b)=1$ ), nên $y: b^2$. Ta đã chọn sao cho $y$ không thể nào có ước chính phương nào ngoài 1 , nên $b=1$ ! Từ đó ta có $\sqrt{d}=a$, hay $d=a^2$, vô lý.


Hướng dẫn giải

Gọi $d>0$ là một ước chung của $a^m+b^n, a^m-b^n$. Khi đó $\left\{\begin{array}{I}2 a^m=\left(a^m+b^n\right)+\left(a^m-b^n\right) \vdots d \\\ 2 b^n=\left(a^m+b^n\right)-\left(a^m-b^n\right) \vdots d\end{array}\right.$.
Để ý rằng $a, b$ khác tính chẵn lẻ, nên $a^m+b^n$ và $a^m-b^n$ luôn lẻ. Nhưng $d$ là một ước chung, nên $d$ lẻ. Như vậy $a^m, b^n: d$.

Nếu $d \neq 1$, gọi $p$ là một ước nguyên tố của $d$ (có thể $d=p$ ). Khi đó $a^m, b^n: p$, nên ta cũng có $a, b: p$. Điều này mâu thuẫn với giả thiết $a, b$ nguyên tố cùng nhau, nên $d=1$. Nhưng $d$ bất kỳ, nên $a^m+b^n, a^m-b^n$ chỉ có ước chung (dương) là 1 . Hay nói cách khác, $a^m+b^n, a^m-b^n$ nguyên tố cùng nhau.

Ví dụ 3.4. Cho 2 số hữu tỷ $\dfrac{a}{b}, \dfrac{c}{d}$ viết ở dạng tối giản (tức $(a, b)=(c, d)=1$ ) sao cho $d\frac{a}{b}+\dfrac{c}{d}$ là một số nguyên. Chứng minh rằng $|b|=|d|$.

Hướng dẫn giải

Ta có $\dfrac{a}{b}+\dfrac{c}{d}=\dfrac{a d+b c}{b d}$ là một số nguyên, nên $a d+b c: b$, hay $a d: b$. Nhưng $a, b$ nguyên tố cùng nhau, nên $d: b$.
Chứng minh tương tự với $a d+b c: d$, ta có $b: d$. Như vậy $|b|=|d|$.

Ví dụ 3.5 (Spanish Mathematical Olympiad 1996).
Giả sử $a, b$ là các số nguyên dương sao cho $\dfrac{a+1}{b}+\dfrac{b+1}{a}$ là số nguyên. Nếu $d$ là ước chung lớn nhất của $a, b$
(a) Chứng minh rằng $a+b \geq d^2$.
(b) Tìm một cặp $(a, b)$ mà $a+b=d^2$.

Hướng dẫn giải

(a) Đặt $a=d a^{\prime}, b=d b^{\prime}$, ta có
$$
\dfrac{a+1}{b}+\dfrac{b+1}{a}=\dfrac{d^2\left(a^{\prime 2}+b^{\prime 2}\right)+d\left(a^{\prime}+b^{\prime}\right)}{d^2 a^{\prime} b^{\prime}} \in \mathbb{Z}
$$
nên $\dfrac{d^2\left(a^{\prime 2}+b^{\prime 2}\right)+d\left(a^{\prime}+b^{\prime}\right)}{d^2}=a^{\prime 2}+b^{\prime 2}+\dfrac{a^{\prime}+b^{\prime}}{d}$ cūng là số nguyên. Như vậy $a^{\prime}+b^{\prime}: d$. Nhưng $a, b$ nguyên dương, nên $a^{\prime}+b^{\prime} \geq d$, hay $a+b=d\left(a^{\prime}+b^{\prime}\right) \geq d^2$.
(b) $a=3, b=6$, thì $\dfrac{a+1}{b}+\dfrac{b+1}{a}=3$ và $a+b=9=\operatorname{gcd}(a, b)^2$.

Ví dụ 3.6 (Romanian Mathematical Olympiad 2003).
Cho $n$ là một số chẵn nguyên dương. Tìm tất cả các số nguyên dương $a, b$ sao cho $a^n+b^n: a+b$.

Hướng dẫn giải

Do $n$ chẵn ta có $a^n-b^n: a^2-b^2: a+b$. Như vậy
$$
\left\{\begin{array}{l}
2 a^n=\left(a^n+b^n\right)+\left(a^n-b^n\right) \vdots a+b \\\
2 b^n=\left(a^n+b^n\right)-\left(a^n-b^n\right) \vdots a+b
\end{array}\right.
$$

Gọi $d=(a, b)$, và $a=d u, b=d v$, ta có $u, v$ nguyên tố cùng nhau và $\operatorname{gcd}(a, b)=2 d^n \operatorname{gcd}\left(u^n, v^n\right)=$ $2 d^n: d(u+v)$. Nói cách khác, $2 d^{n-1}: u+v$.

Để cho ra tất cả giá trị $a, b$ có thể, ta bắt đầu với 2 số $u, v$ nguyên dương và nguyên tố cùng nhau. Tiếp theo chọn $d$ bất kỳ sao cho $2 d^{n-1}: u+v(d$ luôn tồn tại do ta có thể chọn $d=u+v)$. Khi đó $a=d u, b=d v$ thỏa mãn đề bài.

Thật vậy, từ $a^n+b^n=d^n\left(u^n+v^n\right)$, ta chia làm 2 trường hợp
(a) Nếu $u, v$ đều lẻ: ta có $u^n+v^n$ chẵn, nên $a^n+b^n: 2 d^n: d(u+v)=a+b$.
(b) Nếu, không mất tính tổng quát, $u$ chẵn, $v$ lẻ: do $2 d^{n-1}: u+v$, và $u+v$ lẻ, nên $d^{n-1}: u+v$. Từ đó $a^n+b^n: d^n: d(u+v)=a+b$.

Ta kết luận $a=d u, b=d v$, với $u, v$ nguyên tố cùng nhau sao cho $u+v$ là ước của $2 d^{n-1}$.

Ví dụ 3.7 (India Mathematical Olympiad 1998).
Tìm tất cả các bộ số nguyên dương $(x, y, n)$ sao cho
$$
\operatorname{gcd}(x, n+1)=1 \text { và } x^n+1=y^{n+1} .
$$

Hướng dẫn giải

Do $x>0$, nên $y^{n+1}=x^n+1>1$. Ta có
$$
x^n=y^{n+1}-1=(y-1)\left(y^n+y^{n-1}+\cdots+y+1\right)
$$

Do $y-1>1$, ta phải có $y-1: p$ với $p$ là một ước nguyên tố nào đó của $x$. Từ đó
$$
y^n+y^{n-1}+\cdots+y+1 \equiv \underbrace{1+1+\cdots+1}_{n \text { số } 1} \equiv n+1 \quad(\bmod p)
$$

Như vậy $p$ là ước chung của $x$ và $n+1$, vô lý.

Ví dụ 3.8 (Bulgarian Mathematical Olympiad 2001).

Tìm tất cả các bộ $(a, b, c)$ nguyên dương sao cho $a^3+b^3+c^3$ chia hết cho $a^2 b, b^2 c$, và $c^2 a$.

Hướng dẫn giải

Đầu tiên để ý rằng nếu $d$ là ước chung của $a, b$, ta có $a^3+b^3+c^3: a^2 b: d^3$, nên $c: d$. Như vậy nếu ta đặt $d=(a, b, c)$, và $a=d u, b=d v, c=d w, u, v$ phải nguyên tố cùng nhau. Chứng minh tương tự, ta có $u, v, w$ đôi một nguyên tố cùng nhau.

Do $a^3+b^3+c^3: a^2 b$, ta có
$$
d^3\left(u^3+v^3+w^3\right): d^3 u^2 v \Leftrightarrow u^3+v^3+w^3: u^2 v
$$

Từ đó, $u^3+v^3+w^3: u^2$, và $v^3+w^3: u^2$. Chứng minh tương tự, ta cūng có $u^3+v^3+w^3: v^2, w^2$, và $w^3+u^3: v^2, u^3+v^3: w^2$. Nhưng $u, v, w$ nguyên tố cùng nhau đôi một, nên
$$
\left\{\begin{array}{l}
u^3+v^3+w^3: u^2 v^2 w^2 \\\
v^3+w^3: u^2 \\\
w^3+u^3: v^2 \\\
u^3+v^3: w^2
\end{array}\right.
$$

Không mất tính tổng quát, giả sử $u \leq v \leq w$. Do $a, b, c$ nguyên dương, $u, v, w$ cũng nguyên dương, và $u^2 v^2 w^2 \leq u^3+v^3+w^3 \leq 3 w^3$. Nói cách khác, $w \geq \dfrac{u^2 v^2}{3}$. Mặt khác, $u^3+v^3: w^2$, nên ta được
$$
u^3+v^3 \geq w^2 \geq \dfrac{u^4 v^4}{9} (*)
$$

Nhưng $u \leq v$, nên $2 v^3 \geq u^3+v^3 \geq \frac{u^4 v^4}{9}$, hay $u^4 v \leq 18$. Ta suy ra $u=1$ hoặc $u=2$. Nhưng $u=2$ thì $v \geq 2$, nên $32 \leq u^4 v \leq 18$, vô lý.
*Như vậy $u=1$. Nếu $v=1$ thì 2 : $w^2$, cho nên $w=1$. Ta có bộ $(a, b, c)=(d, d, d)$ thỏa mãn. Nếu $v \geq 2$, ta phải có $w>v$, hay $w \geq v+1 \geq 3$ do $v, w$ nguyên tố cùng nhau. Nhưng $u^3+v^3+w^3: u^2 v^2 w^2$, nên ta có
$$
1+v^3+w^3: v^2 w^2 \Rightarrow v^2 w^2 \leq 1+v^3+w^3 \leq 1+(w-1)^3+w^3<2 w^3
$$

Chia $w^2$ cho cả 2 vế, ta được $v^2<2 w$, hay $w>\frac{v^2}{2}$. Mặt khác, ta có $v^3+u^3: w^2$, nên
$$
v^3+1 \geq w^2>\frac{v^4}{4} \Leftrightarrow 4>v^3(v-4)
$$

Vậy $v \leq 4$. Nhưng $v \geq 2$, ta xét các trường hợp sau
(a) $v=4$ : khi đó $u^3+v^3=65: w^2$, nên $w=1$ (vô lý do $v \leq w$ ).
(b) $v=3$ : khi đó $u^3+v^3=28: w^2$, nên $w \in{1,2}$ (cũng vô lý như trên).
(c) $v=2$ : khi đó $u^3+v^3=9: w^2$, nên $w=3$ (do $w \geq v$ ).

Kiểm tra lại, ta nhận $(a, b, c)=(d, 2 d, 3 d)$ và các hoán vị của nó. Ta kết luận
$$
\begin{aligned}
& (a, b, c)=(k, k, k),(k, 2 k, 3 k),(k, 3 k, 2 k), \
& \quad(2 k, k, 3 k),(2 k, 3 k, k),(3 k, k, 2 k),(3 k, 2 k, k) \quad k \geq 1
\end{aligned}
$$

Ví dụ 3.9. Cho các số nguyên dương $x, y, z$ sao cho $\dfrac{1}{x}+\dfrac{1}{y}=\dfrac{1}{z}$. Giả sử $x, y, z$ nguyên tố cùng nhau (tức $(x, y, z)=1$ ), chứng minh rằng $x+y$ là một số chính phương.

Hướng dẫn giải

Viết lại phương trình thành $x+y=\dfrac{x y}{z}$. Đặt $d=(x, y)$, và $x=d a, y=d b$, ta có $(d, z)=1$ (do $(x, y, z)=1)$ và $(a, b)=1$. Thêm nữa
$$
a+b=\dfrac{d a b}{z}
$$

Ta có $d a b: z$, và $(d, z)=1$, nên $a b: z$. Do $(a, b)=1$, ta sẽ chứng minh $z$ có thể tách thành $z=r s$ sao cho $a: r$ và $b: s$.
*Đặt $r=(a, z)$ và $s=(b, z)$.

  • Theo định nghĩa $a: r, b: s$. Nhưng $(a, b)=1$, nên $(r, s)=1$.
  • Tương tự z:r,s. Kết hợp với điều chứng minh ở trên, ta có $z$ :rs.
  • Mặt khác, đặt $a=k r, b=l s$ và $z=q(r s)$, ta có $k l r s: q r s$, nên $k l: q$. Ta cũng có $(a, z)=r$, nên $(k, q s)=1$.

Như vậy $k l: q$ và $(k, q)=1$. Chứng minh tương tự, ta có $(l, q)=1$. Từ đó $q=1$, và $z=r s$.
Tóm tắt lại, ta có $a=k r, b=l s$ và $z=r s$.
*Thế vào $a+b=\dfrac{d a b}{z}$, ta có
$$
k r+l s=d k l
$$

Để ý $(a, b)=1$ nên $(k, l s)=1$. Mặt khác, $l s=d k l-k r: k$, cho nên $k=1$. Chứng minh tương tự, ta có $l=1$, nên $a b=r s=z$, và $a+b=\dfrac{d a b}{z}=d$. Từ đó $x+y=d(a+b)=d^2$ là một số chính phương.

Ví dụ 3.10. Giải phương trình nghiệm nguyên sau (theo các biến $x, y, n, m$ ) với $m, n \geq 0$.
$$
x^n+y^n=2^m
$$

Hướng dẫn giải

Đặt $d=(x, y)>0$ và $x=d u, y=d v$, ta có $u, v$ nguyên tố cùng nhau và $d^n\left(u^n+v^n\right)=2^m$. Như vậy $d=2^e\left(0 \leq e \leq \frac{m}{n}\right)$. Đặt $k=2^{m-n e}$, ta xét phương trình sau (với $u, v$ nguyên tố cùng nhau).
$$
u^n+v^n=2^k
$$
(a) Nếu $n$ chẵn

(a) Nếu $n=0$ : phương trình gốc trở thành $2^m=2$, nên $m=1$. Ta nhận bộ nghiệm $(x, y, 0,1)$ với mọi $x, y \neq 0$.

(b) Nếu $n \geq 2$ :
i. Nếu $k=0$ : ta có $u^n+v^n=1$. Nhưng $n$ chẵn, nên phương trình chỉ có 4 nghiệm $(0, \pm 1)$ và $( \pm 1,0)$. Ta nhận bộ
$$
(x, y, m, n)=\left( \pm 2^e, 0, n e, n\right),\left(0, \pm 2^e, n e, n\right) \quad(n \text { chẵn })
$$
ii. Nếu $k \geq 1$ : ta có $u^n+v^n$ chẵn. Kết hợp với $u, v$ nguyên tố cùng nhau, ta được $u, v$ cùng lẻ. Xét modulo 4, ta có $2^k=u^n+v^n \equiv 1+1 \equiv 2(\bmod 4)$. Nói cách khác $k=1$ và $u^n+v^n=2$, hay $u, v= \pm 1$. Ta nhận bộ
$$
(x, y, m, n)=\left( \pm 2^e, \pm 2^e, n e+1, n\right) \quad(n \text { chẵn })
$$

(b) Nếu $n$ lẻ: ta có $2^k=(u+v)\left(u^{n-1}-u^{n-2} v+\cdots+v^{n-1}\right)$, nên $u+v=2^s$ với $s \geq 0$.
(a) Nếu $n=1$ : ta có $u+v=2^k$, nên ta nhận các bộ sau
$$
(x, y, m, n)=\left(u, 2^m-u, m, 1\right) \quad(u \text { nguyên bất kỳ })
$$

(b) Nếu $n \geq 3$ :
i. Nếu $k=0$ : ta có $u^n+v^n=1$. Nhưng $u^n+v^n=(u+v)\left(u^{n-1}-u^{n-2} v+\cdots-u v^{n-2}+v^{n-1}\right)$, cho nên $u+v= \pm 1$.
*Với $v=1-u$, ta xét phương trình sau
$$
u^n-(u-1)^n=1
$$

Ta có $u=0,1$ là nghiệm, cho nên ta nhận các bộ sau
$$
(x, y, m, n)=\left(2^e, 0, n e, n\right),\left(0,2^e, n e, n\right) \quad(n \text { lẻ })
$$

Nếu $u \geq 2$, ta chứng minh
$$
u^n-(u-1)^n>1
$$
với mọi $n \geq 2$ bằng quy nạp. Khi $n=2$, ta có $u^2-(u-1)^2=2 u-1 \geq 3>1$. Giả sử bất đẳng thức đúng với $n$, ta chứng minh nó đúng với $n+1$
$$
\begin{aligned}
u^{n+1}-(u-1)^{n+1} & =u^n+(u-1) u^n-(u-1)^{n+1} \
& =u^n+(u-1)\left[u^n-(u-1)^n\right] \
& \geq 2^n+(2-1) \cdot 1>1
\end{aligned}
$$

Nếu $u \leq-1$, ta cũng chứng minh $u^n-(u-1)^n=(1-u)^n-(-u)^n>1$ với mọi $n \geq 2$ bằng quy nạp. Khi $n=2$, ta có $(1-u)^n-(-u)^n=-2 u+1>1$. Giả sử bất đẳng thức đúng với $n$, ta chứng minh nó đúng với $n+1$
$$
\begin{aligned}
(1-u)^{n+1}-(-u)^{n+1} & =(1+w)^{n+1}-w^{n+1} \quad(\text { đặt } w=-u \geq 1) \
& =w(w+1)^n+(w+1)^n-w^{n+1} \
& =(w+1)^n+w\left[(w+1)^n-w^n\right] \
& \geq 2^n+1 \cdot 1>1
\end{aligned}
$$
*Với $v=-1-u$, ta xét phương trình sau
$$
u^n-(u+1)^n=1 \Leftrightarrow(u+1)^n-u^n=-1
$$

Dùng những bất đẳng thức ta đã chứng minh ở trên, cộng với trường hợp $u=0,1$ không thỏa, ta kết luận trường hợp này vô nghiệm.
ii. Nếu $k \geq 1$ : ta có $u^n+v^n$ chẵn, và $u, v$ nguyên tố cùng nhau, nên $u, v$ cùng lẻ. Như vậy
$$
u^{n-1}-u^{n-2} v+\cdots-v^{n-1} \equiv \underbrace{1+1+\cdots+1}_{n \text { số } 1} \equiv n \equiv 1 \quad(\bmod 2)
$$

Kết hợp với $(u+v)\left(u^{n-1}-u^{n-2} v+\cdots+v^{n-1}\right)=2^k$, ta phải có

Kết hợp với $(u+v)\left(u^{n-1}-u^{n-2} v+\cdots+v^{n-1}\right)=2^k$, ta phải có
$$
\left\{\begin{array}{l}
u+v=2^k \quad(k \geq 1) \\\
u^{n-1}-u^{n-2} v+\cdots+v^{n-1}=1
\end{array}\right.
$$

Để ý $u^n+v^n=2^k=u+v$, ta sẽ chứng minh

$\left(u^n-v^n\right)(u-v) \geq 0$ với mọi $u, v$ và $n$ lẻ bằng quy nạp lên $n$. Trường hợp $n=1$ chính là $(u-v)^2 \geq 0$, còn $n=3$ là $\left(u^3-v^3\right)(u-v)=(u-v)^2\left(u^2+u v+v^2\right) \geq 0$.

Giả sử nó đúng với $n-2$ và $n$, ta chứng minh nó cũng đúng với $n+2$
$$
\begin{aligned}
\left(u^{n+2}-v^{n+2}\right)(u-v)= & \left(u^{n+2}-u^2 v^n+u^2 v^n-u^n v^2+u^n v^2-v^{n+2}\right)(u-v) \
= & u^2\left(u^n-v^n\right)(u-v)+u^2 v^2\left(u^{n-2}-v^{n-2}\right)(u-v) \
& +v^2\left(u^n-v^n\right)(u-v) \geq 0
\end{aligned}
$$

  • $2\left(u^n+v^n\right) \geq\left(u^2+v^2\right)\left(u^{n-2}+v^{n-2}\right)$ với mọi $u, v$ và $n \geq 3$ lẻ. Thật vậy, bất đẳng thức tương đương với
    $$
    u^n+v^n-u^2 v^{n-2}-v^2 u^{n-2} \geq 0 \Leftrightarrow\left(u^{n-2}-v^{n-2}\right)(u-v) \geq 0
    $$

Đúng theo bất đẳng thức ta đã chứng minh ở trên.
Áp dụng vào bài toán, ta có $u+v=u^n+v^n \geq \frac{u^2+v^2}{2} \cdot\left(u^{n-2}+v^{n-2}\right) \geq\left(\frac{u^2+v^2}{2}\right)^2$. $\left(u^{n-4}+v^{n-4}\right) \geq \cdots\left(\frac{u^2+v^2}{2}\right)^{(n-1) / 2}(u+v)$. Nhưng $u+v=2^k \geq 2^1>1$, nên
$$
\left(\frac{u^2+v^2}{2}\right)^{(n-1) / 2} \leq 1 \Leftrightarrow u^2+v^2 \leq 2
$$

Xét các giá trị $u, v=0, \pm 1$ thỏa mãn điều kiện trên, ta được các cặp $(u, v)=$ $(0,0),( \pm 1,0),(0, \pm 1),( \pm 1, \pm 1)$. Thử vào $u+v=u^n+v^n=2^k$ (với $2^k \geq 2^1=2$ ), ta chỉ có đúng $u=v=1$ và $k=1$ thỏa. Ta nhận các bộ
$$
(x, y, m, n)=\left(2^e, 2^e, n e+1, n\right) \quad(n \geq 3 \text { lẻ })
$$

Tổng hợp các trường hợp lại, ta kết luận các nghiệm $(x, y, m, n)$ như sau
(a) $\left(2^e, 0, n e, n\right),\left(0,2^e, n e, n\right)$, và $\left(2^e, 2^e, n e+1, n\right)(e, n \geq 0)$.
(b) $\left(-2^e, 0, n e, n\right),\left(0,-2^e, n e, n\right)$, và $\left( \pm 2^e, \pm 2^e, n e+1, n\right)(e, n \geq 0, n$ chẵ $)$.
(c) $\left(u, 2^m-u, m, 1\right)(u \in \mathbb{Z}, m \geq 0)$

Ví dụ 3.11. Tìm tất cả các số nguyên dương $x, y, z$ sao cho
$$
16 x y z=d(x+y+z)^2
$$
với $d$ là ước chung của $x, y, z$

Hướng dẫn giải

Đặt $x=d a, y=d b, z=d c$, ta có $(a, b, c)=1$. Phương trình tương đương với $16 a b c=(a+b+c)^2$.
*Gọi $p^{2 k+1}$ là một ước của $a, p$ nguyên tố. Ta sẽ chứng minh $p^{2 k+2}$ cũng là ước của $a$.

(a) Nếu $p=2$ : đặt $a=2^{2 k+1} u$, ta có
$$
2^{2 k+5} u b c=\left(2^{2 k+1} u+b+c\right)^2
$$

Nếu $b$ chẵn thì $c$ cũng phải chẵn (và ngược lại), nhưng điều này mâu thuẫn với $a, b, c$ nguyên tố cùng nhau. Như vậy $b, c$ phải lẻ. Đê ý vế trái là bội của $2^{2 k+5}$ (mũ lẻ), nên $Q^2: 2^{2 k+5}(Q=$ $2^{2 k+1} u+b+c$. Nói cách khác, $Q: 2^{k+3}$ hay $Q=2^{k+3} R$. Từ đó
$$
2^{2 k+5} u b c=Q^2=2^{2 k+6} R^2
$$
nên $2^{2 k+5} u b c: 2^{2 k+6}$. Nhưng $b, c$ lẻ, nên ta có $u: 2$. Như vậy $a=2^{2 k+1} u: 2^{2 k+2}$.
(b) Nếu $p>2$ : lập luận tương tự như trên, ta đặt $Q=a+b+c$ và $a=p^{2 k+1} u$. Phương trình tương đương với
$$
Q^2=16 p^{2 k+1} u b c: p^{2 k+1}
$$
hay $Q: p^{k+2}$. Ta có $16 p^{2 k+1} u b c=Q^2: p^{2 k+2}$. Nhưng $p>2$, nên $u b c: p$.

Giả sử, không mất tính tổng quát b:p. Khi đó $(a+b+c)^2=16 a b c: p$, nên $a+b+c: p$. Nhưng $a: p$, nên c:p. Ta có điều vô lý do $a, b, c$ nguyên tố cùng nhau. Như vậy $u: p$, nên $a=p^{2 k+1} u: p^{2 k+2}$.

Như vậy nếu $a=p_1^{\alpha_1} p_2^{\alpha_2} \cdots p_m^{\alpha_m}$ là phân tích thừa số nguyên tố, các số mũ $\alpha_i$ phải chẵn (nếu $\alpha_i$ lẻ thì $p_i^{\alpha_i+1}$ cũng là ước của $a$, vô lý). Cùng với $a>0$, ta kết luận $a$ là số chính phương. Chứng minh tương tự, $b, c$ cũng chính phương.
*Đặt tiếp $a=u^2, b=v^2, c=w^2(u, v, w>0)$, ta có phương trình
$$
16 u^2 v^2 w^2=\left(u^2+v^2+w^2\right)^2 \Leftrightarrow u^2+v^2+w^2=4 u v w
$$

Do $a, b, c$ nguyên tố cùng nhau, $u, v, w$ cũng phải nguyên tố cùng nhau. Mặt khác, xét modulo 4 cho cả 2 vế, ta có $u^2+v^2+w^2 \equiv 0,1,2,3(\bmod 4)$, với $u^2+v^2+w^2 \equiv 0(\bmod 4)$ khi và chỉ khi $u^2, v^2, w^2 \equiv 0 (\text{b mod 4} )$. Như vậy $u, v, w$ đều chẵn, vô lý.

Ta kết luận phương trình vô nghiệm.

CHUYÊN ĐỀ SỐ HỌC : PHÉP CHIA HẾT VÀ PHÉP CHIA CÓ DƯ

MỘT SỐ VÍ DỤ

 

Ví dụ 1

Cho $a$ là một số nguyên. Tìm UCLN $(2 a+3,3 a+4)$.

Lời giải

Gọi $d=(2 a+3,3 a+4)$, ta có $d \backslash 2 a+3$ và $d \backslash 3 a+4$.

Vì $3(2 a+3)-2(3 a+4)=1$ nên $d$ là ước của 1 hay $d=1$.

Ví dụ 2

Cho $\mathrm{a}, b$ là các số nguyên dương sao cho $a^2+b^2$ chia hết cho tích $a . b$. Hãy tính giá trị của biểu thức

$A=\frac{a^2+b^2}{a b} .$

(Thi học sinh giỏi Toán 9 – Thành phố Hà Nội, năm 2002).

Lời giải

Gọi $d=(a, b)$ thì $a=d . a_1$ và $b=d . b_1$ với $\left(a_1, b_1\right)=1$. Ta có :

$a^2+b^2=d^2\left(a_1^2+b_1^2\right) \text { và } a b=d^2 a_1 b_1 .$

  • Vì $a^2+b^2$ chia hết cho $a b$ nên $a_1^2+b_1^2$ chia hết cho $a_l b_1$. Suy ra $a_1^2+b_1^2$ chia hết cho $a_l$ và $b_l$. Suy ra $a_1^2$ chia hết cho $b_l$ và $b_1^2$ chia hết cho $a_l$.
  • Vì $\left(a_1, b_1\right)=1$ nên $\mathrm{a}_1$ chia hết cho $\mathrm{b}_1$ và $\mathrm{b}_1$ chia hết cho $\mathrm{a}_1$.

Suy ra $a_l=b_1=1$. Vậy,

$A=\frac{d^2\left(a_1^2+b_1^2\right)}{d^2 c_1 b_1}=\frac{2 d^2 a_1^2}{d^2 c_1^2}=2$

Ví dụ 3

Chứng minh rằng với mọi số nguyên dương $n$ ta đều có $n^3+5 n$ chia hết cho 6 .

(Thi vào lớp 10 chuyên, DHKHTN ĐHQGHN năm 1996).

Lời giải

Ta có $n^3+5 n=\left(n^3-n\right)+6 n$. Để chứng minh $n^3+5 n$ chia hết cho 6 ta chứng minh $n^3-n$ chia hết cho 6 .

Do $n^3-n=n(n-1)(n+1)$ là tích của ba số nguyên liên tiếp nên chia hết cho 2 và 3 .

Vì $(2,3)=1$ nên $n^3-n$ chia hết cho tích $2 \times 3=6$.

Ví dụ 4

Cho $a, b, c$ là các số nguyên. Chứng minh rằng $a^3+b^3+c^3$ chia hết cho 6 khi và chỉ khi $a+b+c$ chia hết cho 6 .

Lời giải

Xét $A=a^3+b^3+c^3-a-b-c=\left(a^3-a\right)+\left(b^3-b\right)+\left(c^3-c\right)$.

Theo ví dụ 3 thì $a^3-a \cdot b^3-b$ và $c^3-c$ đều chia hết cho 6 . Suy ra $A$ chia hết cho 6. Vậy. $a^3+b^3+c^3$ chia hết cho 6 khi và chỉ khi $a+b+c$ chia hết cho 6 .

Ví dụ 5

Chứng minh $S=n^2+3 n-38$ không chia hết cho 49 , với mọi số tự nhiên $n$.

Lời giải

Giả sử tồn tại $n$ sao cho $S=n^2+3 n-38$ chia hết cho $+9$. Vì

$n^2-4 n+4=n^2+3 n-38-7(n-6)$

nên $n^2-t n+4$ chia hết cho 7 hay $(n-2)^2$ chia hết cho 7 . Suy ra $n-2$ chia hết cho 7 hay $n=2+7 t$.

Thay vào $S$ ta được : $S=49\left(t^2+t\right)-28$. Suy ra $S$ không chia hết cho 49 , trái với điều giả sử.

Vậy $S$ không chia hết cho 49 với mọi số tự nhiên $n$.

Ví dụ 6

Chứng minh rằng với mọi số tự nhiên $n$ ta luôn có

$A=2005^n+60^n-1897^n-168^n \text { chia hết cho } 2004 \text {. }$

Lời giải

Ta có $2004-12 \times 167$. Vì $(12,167)=1$ nên để chứng minh $A$ chia hết cho 2004 ta chứng minh $A$ chia hết cho 12 và 167 .

Ta có: $A=\left(2005^n-1897^n\right)-\left(168^n-60^n\right)$.

Áp dụng tính chất $a^{\prime \prime}-b^n$ chia hết cho $a-b$ với mọi $n$ tự nhiên và $a-b \neq 0$. ta suy ra $2005^n-1897^n$ chia hết cho $2005-1897=108=12 \times 9$.

Suy ra $2005^n-1897^n$ chia hết cho 12 . Mặt khác, 168 và 60 đều chia hết cho 12 nên $168^n-60^n$ chia hết cho 12 . Vậy $A$ chia hết cho 12 .

Tương tự như trên, ta có

$A=\left(2005^n-168^n\right)-\left(1897^n-60^n\right) .$

Cũng lập luận tương tự như trên, ta có $2005^n-168^n$ chia hết cho $2005-168=1837$; $1897^n-60^n$ chia hết cho $1897-60=1837$ và $1837=11 \times 167$ nên $2005^n-168^n$ và $1897^n-60^n$ chia hết cho 167 . Suy ra $A$ chia hết cho 167 .

Vậy ta có điều phải chứng minh.

BÀI TẬP

1. Chứng minh $a+2 \mathrm{~b}$ chia hết cho 3 khi và chỉ khi $b+2 a$ chia hết cho 3 .

2. Giả sử $a-c$ là ước của $a b+c d$. Chứng minh rằng $a-c$ cũng là ước của $a d+b c$.

3. Cho $a, b \in \mathbb{N}$. Chứng minh $\frac{11 a+2 b}{19} \in \mathbb{Z}$ khi và chỉ khi $\frac{18 a+5 b}{19} \in \mathbb{Z}$.

4. Cho $n$ nguyên dương. Chứng minh rằng

$(n !+1,(n+1) !+1)=1 .$

5. Cho $a, b$ là các số nguyên. Chứng minh rằng

$(5 a+3 b, 13 a+8 b)=(a, b) \text {. }$

6. Cho các số nguyên $m, n, p, q$ thỏa mãn $|p \cdot m-q \cdot n|=1$. Chứng minh rằng với mọi cặp số nguyên $a, b$ ta đều có

$(m a+n b, p a+q b)=(a, b) .$

7. Giả sử $(a, n)=p$ và $(b, n)=q$. Chứng minh rằng $(a b, n)=(p q, n)$.

8. Cho $a \leq b \leq c$ và $b=a q_1+r_1, c=a \cdot q_2+r_2$. Chứng minh rằng

$(a, b, c)=\left(a, r_1, r_2\right) .$

9. Chứng minh rằng với mọi số tự nhiên $n$, các phân số sau là phân số tối giản

(a) $\frac{21 n+4}{14 n+3}$;

(b) $\frac{15 n^2+8 n+6}{30 n^2+21 n+13}$;

(c) $\frac{n^3+2 n}{n^4+3 n^2+1}$.

$(I M O-1959)$.

10. Xác định các giá trị của $n$ để các phân số sau đây là phân số tối giản

(a) $\frac{n+22}{n+3}$

(b) $\frac{3 n+2}{2 n+3}$

(c) $\frac{18 n+3}{21 n+7}$.

11. Xét phân số

$A=\frac{n^2+4}{n+5} .$

Hỏi có bao nhiêu số tự nhiên $n$ trong khoảng từ 1 đến 2005 sao cho phân số $A$ chưa tối giản?

12. Chứng minh rằng với mọi bộ ba số lẻ $a, b, c$ ta đều có

$\left(\frac{a+b}{2}, \frac{b+c}{2}, \frac{c+a}{2}\right)=(a, b, c) .$

13. Cho $\mathrm{a}, \mathrm{b}, c$ là các số nguyên dương. Chứng minh

a) $(a, b, c)=\frac{(a, b, c) a b c}{(a, b)(b, c)(c, a)}$;

b) $[a, b, c]=\frac{(a, b, c)[a, b][b, c][c, a]}{a b c \ldots}$.

14. Cho $a_1, a_2, \ldots, a_n$ là các số nguyên dương và $n>1$. Đặt

$A=a_1 a_2 \ldots a_n, A_i=\frac{A}{a_i}(i=\overline{1, n}) .$

Chứng minh các đẳng thức sau :

a) $\left(a_1, a_2, \ldots, a_n\right)\left[A_1, A_2, \ldots, A_n\right]=A$;

b) $\quad\left[a_1, a_2, \ldots, a_n \mid\left(A_1, A_2, \ldots, A_n\right): A\right.$.

15. Cho $m, n$ là hai số tự nhiên nguyên tố cùng nhau. Hãy tìm ước số chung lớn nhất của hai số $A=m+n$ và $B=m^2+n^2$.

(Thi học sinh gioi Toán toàn quốc lớp 9 năm 1979).

16. Xác định ước số chung lớn nhất của hai số sau :

a) $(7 a+1,8 a+3)$

b) $(11 a+2,18 a+5)$

trong đó $a$ là một số nguyên cho trước.

17. Cho $n$ là một số nguyên dương. Hãy tính bội số chung nhỏ nhất của các số

$n, n+1, n+2 \text {. }$

18. Chứng minh rằng với mọi số nguyên dương $n$ ta có

$[1,2, \ldots, 2 n]=[n+1, n+2, \ldots, n+n]$

19. Cho số nguyên $a$ không chia hết cho 2 và 3 . Chứng minh rằng :

$A: 4 a^2+3 a+5 \text { chia hết cho } 6 \text {. }$

20. Chứng minh rằng $\frac{a}{3}+\frac{a^2}{2}+\frac{a^3}{6} \in \mathbb{Z}, \forall u \in \mathbb{Z}$.

21. Chứng minh rằng $\mathrm{A}(\mathrm{n})=\mathrm{n}^4+6 \mathrm{n}^3+11 \mathrm{n}^2+6 n$ chia hết cho 24 .

(Thi học sinh giỏi Toán toàn quốc – lớp 9 năm 1975)

22. Chứng minh rằng $n^5-n$ chia hết cho 30 , với mọi $n$.

23. Chứng minh rằng $m^3+3 m^2-m-3$ chia hết cho 48 , với mọi $m$ lẻ.

24. Chứng minh rằng $n^{12}-n^8-n^4+1$ chia hết cho 512 , với mọi $n$ lẻ.

25. Chứng minh rằng $A(n)=n^4 \cdots 14 n^3+71 n^2-154 n+120$ chia hết cho 24 , với mọi số tự nhiên $n$.

26. Chứng minh rằng $n^4-4 n^3-4 n^2+16 n$ chia hết cho 384 , với mọi số tự nhiên $n$ chẵn.

(Thi học sinh giỏi toàn quốc – lớp 9 năm 1970)

27. Tìm tất cả các số nguyên dương $n$ sao cho $n^2+9 n-2$ chia hết cho 11 .

28. Tìm tất cả các số nguyên $x$ sao cho : $\left(x^3-8 x^2+2 x\right)$ chia hết cho $x^2+1$.

(Thi vô địch Bun-ga-ri năm 1977)

29. Cho $f(x)=a x^2+b x+c$ thoả mãn : $f(x) \in \mathbb{Z}, \forall x \in \mathbb{Z}$. Hỏi $a, b, c$ có nhất thiết phải là các số nguyên hay không? Tại sao?

(Thi vào lớp 10 chuyên, $Đ H K H T N$ – ĐHQGHN năm 2001)

30. Chứng minh $n^2+n+2$ không chia hết cho 15 , với mọi $n$ thuộc $\mathbb{Z}$.

31. Chứng minh $n^2+3 n+5$ không chia hết cho 121 , với mọi $n$ thuộc $\mathbb{N}$.

32. Chứng minh $9 n^3+9 n^2+3 n-16$ không chia hết cho 343 , với mọi $n$ thuộc $\mathbb{N}$.

33. Chứng minh $4 n^3-6 n^2+3 n+37$ không chia hết cho 125 , với mọi $n$ thuộc $\mathbb{N}$.

34. Cho $a$ và $b$ thuộc $\mathbb{N}$. Chứng minh rằng $5 a^2+15 a b-b^2$ chia hết cho $49 \mathrm{khi}$ và chỉ khi $3 a+b$ chia hết cho 7 .

35. Cho $a, b \in \mathbb{N}$. Chứng minh rằng $2 a+b$ chia hết cho 7 khi và chỉ khi $3 a^2+10 a b-8 b^2$ chia hết cho 49 .

36. Cho $n \in \mathbb{N}$. Chứng minh rằng số $A=5^n\left(5^n+1\right)-6^n\left(3^n+2^n\right)$ chia hết cho 91 .

(Thi vào lớp 10 chuyên, ĐHSPHN năm 1998).

37. Cho $n \in \mathbb{N}$. Chứng minh $6^{2 n}+19^n-2^{n+1}$ chia hết cho 17 .

38. Chứng minh $2^{8 n} \cdot 5^{6 n}-1980^n-441^n+1$ chia hết cho 1979 , với mọi $n$ thuộc $\mathbb{N}$.

39. Chứng minh $118^n-101^n-16^n-1$ chia hết cho 234 , với mọi $n$ lẻ.

40. Chứng minh $11^{n+2}+12^{2 n+1}$ chia hết cho 133 , với mọi $n$ thuộc $\mathbb{N}$.

41. Chứng minh $5^{2 n-1} \cdot 2^{n+1}+3^{n+1} \cdot 2^{2 n-1}$ chia hết cho 38 , với mọi $n$ thuộc $\mathbb{N}^*$.

42. Chứng minh $5^{n+2}+26.5^n+8^{2 n+1}$ chia hết cho 59. với mọi $n$ thuộc $\mathbb{N}$.

43. Tìm số tự nhiên $n$ lớn nhất sao cho $: 29^n$ là ước của 2003 !.

44. Tìm số tự nhiên $k$ lớn nhất sao cho : $(1994 \text { ! })^{1995} \quad \vdots 1995^k$.

(Thi học sinh giỏi Toán toàn quốc – lớp 9. năm 199t).

45. Cho $n$ thuộc $\mathbb{N}$ và $n>3$. Chứng minh rằng nếu $2^n=10 a+b(0<b<10)$ thì tích $a \cdot b$ chia hết cho 6 .

(Thi học sinh giỏi Toán toàn quốc lớp 9 năm 1983).

46. Cho $n$ thuộc $\mathbb{N}, n \geq 1$. Chứng minh $T_n=1^5+2^5+\ldots+n^5$ chia hết cho tổng của $n$ số tự nhiên đầu tiên $S_n=1+2+\ldots+n$.

(Thi vào lớp 10 chuyên $Đ H S P H N$ năm 2001).

47. Tìm $n$ nguyên dương sao cho : $(n-1)$ ! chia hết cho $n$.

(Thi vô địch Hungari năm 1951).

48. Xác định $n$ nguyên dương $(\mathrm{n} \geq 3$ ) sao cho số $A=1.2 .3 \ldots \mathrm{n}$ (tích của $n$ số nguyên dương đầu tiên) chia hết cho $B=1+2+\ldots+n$.

(Thi vào lớp 10 chuyên ĐHKHTN – ĐHQGHN năm 1994).

49. Cho $a$ và $m$ là các số nguyên dương và $a>1$. Chứng minh rằng

$\left(\frac{a^m-1}{a-1}, a-1\right)=(m, a-1) .$

50. Cho $a, m, n$ là các số nguyên dương và $a \neq 1$. Chứng minh rằng $a^n-1 \backslash a^m-1$ khi và chỉ khi $n \backslash m$.

51. Cho $a, m, n$ là các số nguyên dương và $a>1$. Chứng minh rằng

$\left(a^m-1, a^n-1\right)=a^{(m, n)}-1 .$

52. Cho $a, b$ là hai số nguyên dương không nhỏ hơn 2 và nguyên tố cùng nhau. Chứng minh rằng nếu $m, n$ là hai số nguyên dương thỏa mãn $a^n+b^n \backslash a^m+b^m$ thì ta cũng có $n . \mid m$.

53. Cho $a, b, n$ là các số nguyên dương. Biết rằng với mọi số tự nhiên $k \neq b$ ta đều có $k^n-a$ chia hết cho $k-b$. Chứng minh $a=b^n$.

54. Chứng minh rằng tồn tại vô hạn số tự nhiên $n$ sao cho : $4 n^2+1$ chia hết cho cả 5 và 13 .

55. Giả sử $1-\frac{1}{2}+\frac{1}{3}-\ldots+\frac{1}{1319}=\frac{p}{q}$, trong đó $p, q$ là các số nguyên. Chứng minh rằng $p$ chia hết cho 1979.

56. Cho $a_1, a_2, \ldots, a_n \in{1,-1}, n \in \mathbb{N}^*$ và thoả mãn :

$a_1 a_2+a_2 a_3+\ldots+a_n a_1=0 \text {. }$

Chứng minh $n$ chia hết cho 4 .

57. Chứng minh rằng tổng bình phương của $p$ số nguyên liên tiếp ( $p$ là số nguyên tố, $p>3$ ) chia hết cho $p$.

58. Cho số nguyên $a$ không nhỏ hơn 2 . Hỏi có tồn tại hay không số tự nhiên $A$ sao cho

$a^{2001}<A<a^{2002}$

và $A$ có ít nhất 600 chữ số 0 ở tận cùng?

59. Có tồn tại hay không 4004 số nguyên dương sao cho tổng của 2003 số bất kì đều không chia hết cho 2003 .

(Balkan 2003).

60. Tìm một cặp số nguyên dương $(a, b)$ thoả mãn đồng thời các điều kiện sau :

a) $a b(a+b)$ không chia hết cho 7 .

b) $(a+b)^7-a^7-b^7$ chia hết cho $7^7$.

(IMO-198t).

61. Giả sử $a, b$ là hai số nguyên dương khác nhau. Chứng minh rằng tồn tại vô số số tự nhiên $n$ sao cho $a+n$ và $b+n$ là hai số nguyên tố cùng nhau.

 

LỜI GIẢI – HƯỚNG DẪN – ĐÁP SỐ

1. Suy ra từ đẳng thức : $(a+2 b)+(b+2 a)=3(a+b)$.

2. Suy ra từ đẳng thức : $(a b+c d)-(a d+b c)=(a-c)(b-d)$.

3. Suy ra từ đẳng thức : $5 \cdot \frac{11 a+2 b}{19}-2 \cdot \frac{18 a+5 b}{19}=a$.

4. Giả sử $d=(n !+1,(n+1) !+1)$.

Ta có $d \backslash n !+1$ và $d \backslash(n+1) !+1$ nên $d \backslash(n+1) !+1-n !-1=n ! . n\quad\quad(1)$.

Vì $d \backslash n !+1$ nên $(d, n)=(d, n !)=1$. Từ (1) suy ra $d=1$.

5. Giả sử $d=(a, b)$ và $d^{\prime}=(5 a+3 b, 13 a+8 b)$.

Vì $d \backslash a$ và $d \backslash b$ nên $d \backslash 5 a+3 b$ và $d \backslash 13 a+8 b$. Suy ra $d \backslash d\quad(1)$.

Vì $d^{\prime} \backslash 5 a+3 b$ và $d^{\prime} \backslash 13 a+8 b$ nên

$d^{\prime} \backslash 8(5 a+3 b)-3(13 a+8 b)=a$

và $\quad d^{\prime} \backslash 5(13 a+8 b)-13(5 a+3 b)=b$.

Suy ra $d^{\prime} \backslash d\quad\quad(2)$.

Từ (1) và (2) ta suy ra $d^{\prime}=d$.

6. Giải tương tự bài $1.5$.

7. Ta có $(a, n)=p$ nên $a=p \cdot a_1, n=p n_1$ với $\left(a_1, n_1\right)=1$. Suy ra

$(a b, n)=\left(p a_1 b, p n_1\right)=p \cdot\left(a_1 b, n_1\right)=p\left(b, n_1\right)=(p b, n)$

$\text { Vì }(b, n)=q \text { nên } b=q \cdot b_1 \text { và } n=q \cdot n_2 \text { với }\left(b_1, n_2\right)=1 . \text { Suy ra }$

$(p b, n)=\left(p \cdot q \cdot b_1, q \cdot n_2\right)=q\left(p b_1, n_2\right)=q\left(p, n_2\right)=(p q, n)$

8. Giải tương tự bài $1.5$.

9. a) Giả sử $(21 n+4,14 n+3)=d(d \geq 1)$.

Ta có $d \backslash 21 n+4$ và $d \backslash 14 n+3$ nên $d \backslash 3(14 n+3)-2(21 n+4)=1$.

Vậy $d=1$.

Các bạn tự giải các câu b) và c).

10. a) Ta có $\frac{n+22}{n+3}=1+\frac{19}{n+3}$. Phân số đã cho tói gian khi và chi khi $(n+3,19)=1$ hay $n \neq 19 m-3$.

b) Vì $(2 n+3,2)=1$ nên phân số đã cho tối gian khi và chi khi phân số sau tối gịản

$B=\frac{2(3 n+2)}{2 n+3}=3-\frac{5}{2 n+3} .$

Phân số $B$ tối giann khi và chi khi $(2 n+3,5)=1$.

Ta có $(2 n+3,5) \neq 1$ khi và chi khi $5 \backslash 2 n+3$ hay $2 n+3=5 a$.

Xét $2 n+3=5 a$, ta có $n=2 a+\frac{a-3}{2}$.

Vì $n$ và $a$ là các số nguyên nên $a-3=2 m$, từ đó có $n=5 m+6$.

Vậy phân số đã cho tối giản khi và chi khi $n \neq 5 m+6$.

c) Đáp số: $n \neq 7 m+1$.

11. Giả sử $A$ là phân số chưa tối giản. Đặt $d=\left(n^2+4, n+5\right)$ suy ra $d>1$. Ta có

$d \backslash(n+5)^2-\left(n^2+4\right)=10 n+21=10(n+5)-29$

nên $d \backslash 29$ suy ra $d=29$.

Ngược lại, nếu $n+5$ chia hết cho 29 thì có thể đặt

$n+5=29 . m\left(m \in \mathbb{N}^*\right)$

12. Giải tương tự bài 5 .

13. Giải tương tự bài 5 .

14. Giải tương tự bài 5 .

15. Giả sử $d=(A, B)(d \geq 1)$. Ta có $d \backslash A^2-B$ suy ra $d \backslash 2 m n\quad(1)$.

Vì $d \backslash A$ nên $d \backslash 2 n \cdot A$ hay $d \backslash 2 m n+2 n^2$. Suy ra $d \backslash 2 n^2\quad(2)$.

Tương tự ta cũng có $d \backslash 2 m^2\quad(3)$

Vì $(m, n)=1$ nên $m, n$ không cùng chẵn. Xét các trường hợp:

  • Nếu $m, n \cdot$ khác tính chã̃n lẻ thì $d$ lẻ. Từ (2) và (3) ta suy ra $d \backslash m^2$ và $d \backslash$ $n^2$. Vì $(m, n)=1$ nên $d=1$.

  • Nếu $m, n$ cùng lẻ thì $d$ chã̃n. Đặt $d=2 d$, từ (2) và (3) ta suy ra $d \backslash m^2$ và $d^n \backslash n^2$. Vì $(m, n)=1$ nên $d^n=1$. Suy ra $d=2$.

16. a) Đặt $d=(7 a+1,8 a+3)$.

Ta có $d \backslash 7(8 a+3)-8(7 a+1)=13$ nên $d=1$ hoặc $d=13$.

Để $d=13$ thì điều kiện cần và đủ là $13 \backslash 7 a+1$.

Xét phương trình: $7 a+1=13 x$.

Ta có $a=2 x-\frac{x+1}{7}$ là một số nguyên nên $7 \backslash x+1$.

Đặt $x+1=7 m$ ta được $a=13 m-2, m \in \mathbb{Z}$.

Vậy, khi $a=13 m-2, m \in \mathbb{Z}$ thì $(7 a+1,8 a+3)=13$,

$a \neq 13 m-2, m \in \mathbb{Z}$ thì $(7 a+1,8 a+3)=1$

b) Giải tương tự câu a).

Đáp Số:

  • Nếu $a=19 m-14, m \in \mathbb{Z}$ thì $(11 a+2,18 a+5)=19$

  • Nếu $a \neq 19 m-14, m \in \mathbb{Z}$ thì $(11 a+2,18 a+5)=1$.

18. Giả sử $m=[1,2, \ldots, 2 n]$ và $m^{\prime}=[n+1, \ldots, n+n](n \geq 2)$.

Để chứng minh $m=m^{\prime}$ ta chứng minh $m \backslash m^{\prime}$ và $m^{\prime} \backslash m$.

Vì $n+1, n+2, \ldots, n+n$ là ước của $m$ nên $m^{\prime} \backslash m$.

Ngược lại, xét số $a \in{1,2, \ldots, n}$ tùy ý.

Trong $a$ số nguyên liên tiếp $n+1, \ldots, n+a$ luôn có một số chia hết cho $a$ nên $a \backslash m^{\prime}$. Suy ra các số $1,2, \ldots, 2 n$ đều là ước của $m^{\prime}$ hay $m \backslash m^{\prime}$.

Vậy $m=m^{\prime}$.

19. Vì $a$ không chia hết cho 2 và 3 nên $a$ có dạng: $a=6 m \pm 1(m \in \mathbb{Z})$

  • Với $a=6 m+1$ ta có $A=4(6 m+1)^2+3(6 m+1)+5$

$=6\left(24 m^2+11 m+2\right) \vdots 6 \text {. }$

  • Với $a=6 m-1$ ta có $A=4(6 m-1)^2+3(6 m-1)+5$

$=6\left(24 m^2-5 m+1\right) \vdots 6 \text {. }$

Vậy $A$ chia hết cho 6 , với mọi $a$ không chia hết cho 2 và 3 .

20. Ta có $\frac{a}{3}+\frac{a^2}{2}+\frac{a^3}{6}=\frac{a(a+1)(a+2)}{6}$.

Vì $a(a+1)(a+2)$ là tích của ba số nguyên liên tiếp nên chia hết cho 6 từ đó suy ra đpcm.

21. Ta có $A(n)=n(n+1)(n+2)(n+3)$.

Vì tích của ba số nguyên liên tiếp chia hết cho 3 nên $A(n)$ chia hết cho 3 . Trong bốn số nguyên liên tiếp luôn có hai số chẵn liên tiếp, một trong hai số đó chia hết cho 4 nên $A(n)$ chia hết cho 8 .

Vì $(3,8)=1$ nên $A(n)$ chia hết cho $3 \times 8=24$.

22. Ta có $30=6 \times 5$. Vì $(6,5)=1$ nên để chứng minh $n^5-n$ chia hết cho 30 ta chứng minh $n^5-n$ chia hết cho 6 và 5 .

Ta có $n^5-n=(n-1) n(n+1)\left(n^2+1\right)$. Vì $(n-1) n(n+1)$ là tích ba số nguyên liên tiếp nên chia hết cho 2 và 3 .

Suy ra $n^5-n$ chia hết cho $2 \times 3=6$.

Mặt khác ta lại có

$n^5-n=(n-1) n(n+1)(n^2-4+5)$

$=(n-2)(n-1) n(n+1)(n+2)+5(n-1) n(n+1) .$

Vì $(n-2)(n-1) n(n+1)(n+2)$ là tích cua năm sổ nguyên liên tiếp nên chia hết cho 5 .

Suy ra $n^5-n$ chia hết cho 5 .

Vậy $n^5-n$ chia hết cho 30 .

23. Đặt $A=m^3+3 m^2-m-3$.

Ta có $A=(m+3)\left(m^2-1\right)=(m+3)(m+1)(m-1)$.

Vì $m$ lẻ nên $m=2 n+1(n \in \mathbb{Z})$, từ đó suy ra $A=8 .(n+2)(n+1) n \Rightarrow$ đpcm.

24. Đặt $A=n^{12}-n^8-n^4+1$. Ta có

$A=\left(n^4-1\right)\left(n^8-1\right)=\left[\left(n^2-1\right)\left(n^2+1\right)\right]^2\left(n^4+1\right) .$

Vì $n$ lẻ nên $n=2 m+1$, suy ra $A=64 \cdot[m(m+1)]^2\left(2 m^2+2 m+1\right)^2\left(n^4+1\right)$.

25. Ta có $24=3 \times 8$. Để chứng minh $A(n)$ chia hết cho 24 ta chứng minh $A(n)$ chia hết cho 3 và 8 .

Ta có $A(n)=(n-2)(n-3)(n-4)(n-5)$ (bạn đọc tự phân tích).

Vì $A(n)$ là tích của bốn số nguyên liên tiếp nên $A(n)$ chia hết cho 3 .

Trong bốn số nguyên liên tiếp $n-2, n-3, n-4, n-5$ luôn có hai số chã̃n liên tiếp. Một trong hai số đó chia hết cho 4 , số còn lại chia hết cho 2 nên $A(n)$ chia hết cho 8 . Vì $(3,8)=1$ nên $A(n)$ chia hết cho $3 \times 8=24$.

26. Đặt $A=n^4-4 n^3-4 n^2+16 n$. Ta có $A=n(n-4)\left(n^2-4\right)$.

Vì $n$ chẵn nên $n=2 m(m \in \mathbb{Z})$. Từ đó suy ra $A=16 .(m-2)(m-1) m(m+1)$.

Vì $(m-2)(m-1) m(m+1)$ là tích của 4 số nguyên liên tiếp nên chia hết cho 8 và 3 .

Từ đó có đpcm.

27. Đáp số: $n=11 m+6$ hoặc $n=11 m+7(m \in \mathbb{N})$.

Hướng dẫn :

$\text { Ta có } n^2+9 n-2 \vdots 11 \Leftrightarrow n^2-2 n-2 \vdots 11 \Leftrightarrow 4\left(n^2-2 n-2\right) \vdots 11$

$\Leftrightarrow 4 n^2-8 n+3 \vdots 11 \Leftrightarrow(2 n-1)(2 n-3) \vdots 11 .$

28. Đáp số: $x \in{-8,0,2}$.

Giả sử $\left(x^3-8 x^2+2 x\right) \vdots\left(x^2+1\right)$ suy ra

$x\left(x^2+1\right)-8\left(x^2+1\right)+x+8 \vdots\left(x^2+1\right) \text {. }$

hay $x+8 \vdots\left(x^2+1\right)\quad\quad( * )$

  • Nếu $x+8=0$ thì $x=-8$, thỏa mãn điều kiện đề bài.

  • Nếu $x \neq-8$ thì tù $\left(^*\right)$ ta phải có $|x+8| \geq x^2+1\quad\quad(1)$.

Bất phương trình (1) cho ta $x \in{-2,-1,0,1,2,3}$

Thử trực tiếp ta được $x=0$ và $x=2$ thỏa mãn.

Cách 2

$\text { Ta có } x+8 \vdots\left(x^2+1\right) \Rightarrow x^2+8 x \vdots\left(x^2+1\right) \Rightarrow 8 x-1 \vdots\left(x^2+1\right)$

$\Rightarrow  8(x+8)-(8 x-1) \vdots x^2+1 \Rightarrow 65 \vdots\left(x^2+1\right)$

$\Rightarrow x^2+1 \text { là ước dương của } 65$

$\Rightarrow x^2+1 \in{1,5,13,65} .$

29. Cho $x=0$ suy ra $f(0)=c \in \mathbb{Z}$. Các số $a, b$ không nhất thiết phải là các số nguyên.

Ví dụ, chọn $a=b=\frac{1}{2}$, ta có

$f(x)=\frac{x(x+1)}{2}+c \in \mathbb{Z}, \forall x \in \mathbb{Z} \text {. }$

30. Giả sử $n^2+n+2 \vdots 15$ ta có $n^2+n+2 \vdots 3\quad\quad(1)$.

Từ (1) suy ra $n$ không chia hết cho 3 .

Vậy $n$ có dạng $3 k+1$ hoặc $3 k-1(k \in \mathbb{Z})$, ta có

$n^2-1=(n-1)(n+1) \vdots 3$

$\Rightarrow n^2+n+2=\left(n^2-1\right)+n+3 \text { không chia hết cho } 3 \text {, mâu thuẫn với (1). }$

31. Giả sử $n^2+3 n+5 \vdots 121$ suy ra $n^2+3 n+5 \vdots 11$ hay $4 n^2+12 n+20 \vdots 11$ Vậy

$4 n^2+12 n+9 \vdots 11 \Rightarrow(2 n+3)^2 \vdots 11 \Rightarrow 2 n+3 \vdots 11$

Nhưng khi đó

$4\left(n^2+3 n+5\right)=(2 n+3)^2+11$ không chia hết cho 121 , mâu thuẫn với điều giả sử trên, từ đó suy ra đpcm.

32. Giải tương tự bài 31

33. Giải tương tự bài 31

34. $\Rightarrow \text { ) Giả sử } 5 a^2+15 a b-b^2 \vdots 49 \Rightarrow 5 a^2+15 a b-b^2 \vdots 7$

$\Rightarrow 9 a^2+6 a b+b^2 \vdots 7 \Rightarrow(3 a+b)^2 \vdots 7 \Rightarrow 3 a+b \vdots 7 \text {. }$

$\Leftrightarrow) \text { Giả sử } 3 a+b \vdots 7 \Rightarrow 3 a+b=7 c(c \in \mathbb{Z}) \Rightarrow b=7 c-3 a$

$\Rightarrow 5 a^2+15 a b-b^2=5 a^2+15 a(7 c-3 a)-(7 c-3 a)^2$

$=49\left(c^2+3 a c-a^2\right) \vdots 49 .$

35. Giải tương tự bài 34.

36. Ta có $91=7 \times 13$. Vì $(7,13)=1$ nên để chứng $\operatorname{minh} A \vdots 91$ ta chi cần chứng $\operatorname{minh} A \vdots 7$ và $A \vdots 13$.

  • Chứng $\operatorname{minh} A \vdots$ 7: Ta viết $A$ dưới dạng: $A=\left(25^n-18^n\right)-\left(12^n-5^n\right)$. Vì $\left(25^n-18^n\right) \vdots 25-18=7$ và $\left(12^n-5^n\right) \vdots 12-5=7$ nên $A \vdots 7$.

  • Chứng $\operatorname{minh} A \vdots$ 13: Ta viết $A$ dưới dạng: $A=\left(25^n-12^n\right)-\left(18^n-5^n\right)$ Vì $\left(25^n-12^n\right) \vdots 25-12=13$ và $\left(18^n-5^n\right) \vdots 18-5=13$ nên $A \vdots 13$. Vậy $A \vdots 91, \forall n \in \mathbb{N}$.

37. Đặt $A(n)=6^{2 n}+19^n-2^{n+1}$.

Ta có $A=36^n+19^n-2 \cdot 2^n=\left(36^n-2^n\right)+\left(19^n-2^n\right)$.

Vì $36^n-2^n \vdots 34(=36-2)$ nên $36^n-2^n \quad \vdots 17$ và $19^n-2^n \vdots 17(=19-2)$ nên $A(n) \vdots 17$.

38. Đặt $A(n)=2^{8 n} \cdot 5^{6 n}-1980^n-441^n+1$.

Ta có $A=\left(4000000^n-441^n\right)-(1980-1)^n$.

Vì $\left(4000000^n-441^n\right) \vdots 3999599(=4000000-441=2021)$ nên

$\left(4000000^n-441^n\right) \vdots 1979$

và $1980^n-1 \vdots 1979(=1980-1)$ nên $A(n) \vdots 1979$.

39. Giải tương tự bài $36$.

40. Giải tương tự bài $37$.

41. Giải tương tự bài 36.

42. Giải tương tự bài 37.

43. Các số chia hết cho 29 trong khoảng từ 1 đến 2003 là:

$29 \times 1,29 \times 2,29 \times 3, \ldots, 29 \times 69 \text {. }$

Suy ra $2003 !=29^{69} \cdot 69 ! . A$, trong đó $(A, 29)=1$.

Các số chia hết cho 29 trong khoảng từ 1 đến 69 là: $29 \times 1,29 \times 2$.

Suy ra: $69 !=29^2 \cdot 2 ! . B$, trong đó $(B, 29)=1$.

Vậy $2003 !=29^{71} \cdot 2 \cdot A \cdot B$, trong đó $(A \cdot B, 29)=1$.

Từ đó suy ra $n$ cần tìm là 71 .

44. Đáp số: $k=217455$.

$1995=3 \times 5 \times 7 \times 19 .$

Ta :cần tìm số mũ lớn nhất của 19 trong phân tích tiêu chuẩn của số $(1994 !)^{1995}$. Xem lại bài 43.

45. Ta có $2^n=10 a+b$ nên $b \vdots 2$ hay $a b \vdots 2$. Ta chứng minh $a b \vdots 3$ :

Từ đẳng thức $2^n=10 a+b$ suy ra $2^n$ có chữ số tận cùng là $b$.

Đặt $n=4 k+r(k, r \in \mathbb{N}, 0 \leq r \leq 3)$ ta có $2^n=16^k \cdot 2^r$.

Nếu $r=0$ thì $2^n=16^k$ có chữ số tận cùng là 6 suy ra $b=6$ hay $a b \vdots 6$.

Nếu $1 \leq r \leq 3$ thì $2^n-2^r=2^r\left(16^k-1\right) \vdots 10$ suy ra $2^n$ có tận cùng là $2^r$.

Vậy ta có $b=2^r$, từ đó suy ra

$10 a=2^n-2^r=2^r\left(16^k-1\right) \vdots 3 \Rightarrow a \vdots 3 \Rightarrow a b \vdots 3 .$

46. Ta có $2 S_n=n(n+1)$.

Mặt khác, sử dụng tính chất $a^n+b^n \vdots(a+b), \forall a, b \in \mathbb{N}^*$ và $n$ lẻ ta có

$2 T_n=\left(1^5+n^5\right)+\left(2^5+(n-1)^5\right)+\ldots+\left(n^5+1\right) \vdots(n+1)\quad\quad(1) .$

$2 T_n=\left(1^5+(n-1)^5\right)+\left(2^5+(n-2)^5\right)+\ldots+\left((n-1)^5+1\right)+2 n^5 \vdots n\quad\quad(2) .$

Do $(n, n+1)=1$, từ $(1)$ và $(2)$ ta suy ra

$2 T_n \vdots n(n+1)=2 S_n \Rightarrow T_n \vdots S_n .$

Tổng quát, ta có thể chứng minh được:

$1^k+2^k+\ldots+n^k$ chia hết cho $1+2+\ldots+n, \forall n, k \in \mathbb{N}, n \geq 1$ và $k$ lẻ.

47. Dễ thấy $n=1$ thỏa mãn và $n=4$ không thỏa mãn. Xét $n>1$ và $n \neq 4$ :

Từ giả thiết suy ra $n$ là hợp số, như vậy $n$ có thể viết được dưới dạng $n=p . q$, trong đó $p, q$ là các số nguyên dương thỏa mãn: $2 \leq p, q \leq\left[\frac{n}{2}\right]$.

  • Nếu $p \neq q$ thì trong tích $(n-1) !=1.2 \ldots n$ chứa cả hai số $p$ và $q$ nên $(n-1)$ ! chia hết cho $n$.

  • Nếu $p=q$ thì $p, q>2$ và trong tích $(n-1)$ ! chứa cả $p$ và $2 p$ nên $(n-1)$ ! chia hết cho $n$.

48. Xem lời giải bài 47.

49. Giả sử $d=\left(\frac{a^m-1}{a-1}, a-1\right)$ và $d^{\prime}=(m, a-1)$. Ta có

$\frac{a^m-1}{a-1}=a^{m-1}+a^{m-2}+\ldots+a+1$

$=\left(a^{m-1}-1\right)+\left(a^{m-2}-1\right)+\ldots+(a-1)+m .$

Vì $a^i-1$ chia hết cho $a-1$ và do đó chia hết cho $d$ với mọi $i \geq 1$ nên $d \backslash m$. Suy ra $d \backslash d$.

Ngược lại, vì $d^{\prime} \backslash a-1$ nên $d^{\prime} \backslash a^i-1$ với mọi $i \geq 1$. Hơn nữa, $d^{\prime} \backslash m$ nên $d^{\prime} \backslash$ $\frac{a^m-1}{a-1}$ và do đó $d^{\prime} \backslash d$.

Vậy $d=d$.

50. $\Rightarrow)$ Giả sử $a^n-1 \backslash a^m-1$. Ta chứng minh $n \backslash m$.

Đặt $m=q n+r, 0 \leq r<n$. Ta có

$a^m-1=a^{n q+r}-1=a^r\left(a^{n q}-1\right)+a^r-1$

$\text { Vì } a^n-1 \backslash a^m-1 \text { và } a^n-1 \backslash a^{n q}-1 \text { nên } a^n-1 \backslash a^r-1 .$

$\text { Mặt khác } 0 \leq a^r-1<a^n-1 \text { nên } a^r-1=0 \text { hay } r=0 .$

$\Leftarrow)$ Dễ dàng chứng minh được.

51. Giả sử $d=(m, n)$ và $d=\left(a^m-1, a^n-1\right)$. Đặt $m=d . m_1, n=d . n_1$ ta có $a^m-1=\left(a^d\right)^{m_1}-1$ chia hết cho $a^d-1$

và $\quad a^n-1=\left(a^d\right)^{n_1}-1$ chia hết cho $a^d-1$

nên d’ chia hết cho $a^d-1\quad\quad(1)$.

Mặt khác, vì $d=(m, n)$ nên tồn tại hai số nguyên dương $x, y$ sao cho $m x-n y=d$. Vì $d^n \backslash a^m-1$ và $d^{\prime} \backslash a^n-1$ nên $d^{\prime} \backslash a^{m x}-1$ và $d^n \backslash a^{n y}-1$.

Suy ra $d^n \backslash a^{m x}-a^{n y}=a^{n y}\left(a^d-1\right)$. Vì $\left(d^n, a^{n y}\right)=1$ nên $d^n \backslash a^d-1\quad\quad(2)$.

Từ (1) và (2) ta có đpcm.

52. Giả sử $m$ không chia hết cho $n$, tức là $m=q \cdot n+r$ với $0<r<n$. Ta có

$a^m+b^m=a^{m-n}\left(a^n+b^n\right)-b^n\left(a^{m-n}-b^{m-n}\right) .$

Suy ra $a^n+b^n \backslash a^{m-n}-b^{m-n}$.

Nếu $q-1>0$, lại làm tương tự như trên tai có

$a^{m-n}-b^{m-n}=a^{m-2 n}\left(a^n+b^n\right)-b^n\left(a^{m-2 n}+b^{m-2 n}\right) .$

Suy ra $a^n-b^n \backslash a^{m-2 n}+b^{m-2 n}$.

Cứ lặp lại cách làm trên ta suy ra $a^n+b^n \backslash a^{m-n \cdot k}+(-1)^k b^{m-n k}, \forall k \leq q$.

Đặc biệt với $k=q$ ta có $a^n+b^n \backslash a^r+(-1)^q b^r$. Điều này không xảy ra vì

$0<\left|a^r+(-1)^q b^r\right|<a^r+b^r<a^n+b^n \text {. }$

53. Ta có $k-b \backslash k^n-a=\left(k^n-b^n\right)+\left(b^n-a\right)$ và $k-b \backslash k^n-b^n$ nên $k-b \backslash b^n-a$. Vì điều này đúng với mọi $k$ nên chọn $k$ sao cho $k-b>\left|b^n-a\right|$. Vì $b^n-a$ chia hết cho $k-b$ nên $b^n-a=0$ hay $a=b^n$.

54. Cần tìm $n$ sao cho $4 n^2+1$ chia hết cho 65 . Đặt $n=65 k+r$, ta chọn $r$ sao cho $4 r^2+1=65$ hay $r=\pm 4$.

Khi đó, mọi số $n$ có dạng $65 k \pm 4$ đều thỏa mãn.

55. Ta có

$\text { Ta có } \frac{p}{q}=\left(1+\frac{1}{2}+\ldots+\frac{1}{1319}\right)-2\left(\frac{1}{2}+\frac{1}{4}+\ldots+\frac{1}{1318}\right)$

$=\left(1+\frac{1}{2}+\ldots+\frac{1}{1319}\right)-\left(1+\frac{1}{2}+\frac{1}{3}+\ldots+\frac{1}{659}\right)$

$=\frac{1}{660}+\ldots+\frac{1}{1319}$

$\Rightarrow 2 \cdot \frac{p}{q}=\left(\frac{1}{660}+\frac{1}{1319}\right)+\left(\frac{1}{661}+\frac{1}{1318}\right)+\ldots+\left(\frac{1}{1319}+\frac{1}{660}\right)=\frac{1979 . A}{B} \text {. }$

Từ đó suy ra $p$ chia hết cho 1979 .

56. Đặt $x_1=a_1 \cdot a_2, x_2=a_2 \cdot a_3, \ldots, x_n=a_n \cdot a_1$.

Vì $a_1, a_2, \ldots, a_n \in{-1,1}$ nên $x_1, x_2, \ldots, x_n \in{-1,1}$.

Ta có $x_1+x_2+\ldots+x_n=0$ suy ra trong các số $x_1, x_2, \ldots, x_n$ số các số 1 bằng số các số $-1$.

Giả sử số các số 1 là $m\left(\mathrm{~m} \in \mathbb{N}^*\right)$ thì $n=2 m$ và

$x_1 x_2 \ldots x_n=(-1)^m \text {. }\quad\quad(1)$

Mặt khác, $x_1 x_2 \ldots x_n=\left(a_1 a_2 \ldots a_n\right)^2=1\quad\quad(2)$.

Từ (1) và (2) suy ra $m$ chã̃n và điều đó có nghĩa là $n$ chia hết cho 4 .

57. Giả sử $p$ số nguyên liên tiếp đó là: $a+1, a+2, \ldots, a+p(\mathrm{a} \in \mathbb{Z})$.

Đặt $A=(a+1)^2+(a+2)^2+\ldots+(a+p)^2$. Ta có

$\mathrm{A}=p \cdot a^2+2(1+2+\ldots+p) \cdot a+\left(1^2+2^2+\ldots+p^2\right)$

Mặt khác: $1+2+\ldots+p=\frac{p(p+1)}{2}, 1^2+2^2+\ldots+p^2=\frac{p(p+1)(2 p+1)}{6}$.

Suy ra $6 A=p\left[6 a^2+6(p+1) a+(p+1)(2 p+1)\right]$ chia hết cho $p$.

Do $p$ là số nguyên tố và $p>3$ nên $(p, 6)=1$. Vậy $A$ chia hết cho $p$.

58. Vì $a^{2001}-a^{2000}=a^{2000}(a-1) \geq 2^{2000}=1024^{200}>10^{600}$ nên giữa $a^{2000}$ và $a^{2001}$ có ít nhất $10^{600}$ số nguyên dương liên tiếp. Trong số đó, tồn tại một số chia hết cho $10^{600}$, đó chính là số $A$ cần tìm.

59. Đáp số: 

Tồn tại. Có thể chọn 2002 số chia hết cho 2003 và 2002 số chia cho 2003 dư 1 .

60. Ta có $(a+b)^7-a^7-b^7=7 a b(a+b)\left(a^2+a b+b^2\right)^2$.

Chọn $b=1$ và $a^2+a+1=7^3$ (bạn đọc tự tính $a$ ) ta có đpcm.

61. Không mất tính tổng quát, giả sử $c=a-b>0$.

Ta có $b=q c+r$, với $q \geq 0,0 \leq r<c, q$ và $r$ không đồng thời bằng 0 .

Các số $n$ có dạng $n=c+1-r+k c, k \in \mathbb{Z}$ đều thoả mãn.

 

 

 

 

 

 

 

 

 

 

 

 

 

 

 

 

 

 

 

 

 

 

 

 

 

 

 

 

 

 

 

 

 

 

 

 

 

 

 

 

 

 

 

 

 

 

 

 

 

 

 

 

 

 

 

 

 

 

 

 

 

 

 

 

 

 

 

 

 

 

 

 

 

 

 

 

 

 

 

 

 

 

Bài tập số học ôn thi vào lớp 10 – Phần 3

Bài 21. Chứng minh rằng với mọi số tự nhiên $n > 1$ thì $n^5 + n^4 + 1$ không là số nguyên

Lời giải

$n^5 + n^4 + 1 = n^5+n^4+n^3-n^3+1 = n^3(n^2+n+1) -(n-1)(n^2+n+1) = (n^2+n+1)(n^3-n+1)$
Mà $n^3-n+1 > 1, n^2+n+1>1$ với mọi $n>1$ nên $n^5+n^4+1$ không là số nguyên tố.

Bài 22. Tìm tất cả các số tự nhiên n sao cho ${5^{{5^{n + 1}}}} + {5^{{5^n}}} + 1$ là một số nguyên tố.

Lời giải

Đặt $m = 5^n$ ta có bài trên.

Bài 23. Tìm số nguyên tố $p$ để $p^2 + 2^p$ cũng là số nguyên tố.

Lời giải

Nhận thấy $p=3$ thỏa đề bài.
Xét $p>3$ thì $p$ lẻ và $p$ không chia hết cho 3.
Khi đó $p^2 \equiv 1 (\mod 3)$ và $2^p \equiv -1 (\mod 3)$. Do đó $p^2 + 2^p \equiv 3$ nên không là số nguyên tố.

Bài 24. Cho $p, q$ là các số nguyên tố và phương trình $x^2 – px+q=0$ có nghiệm nguyên dương. Tìm $p$ và $q$.

Lời giải

Gọi $x_1, x_2$ là nghiệm của phương trình. Ta có $x_1 + x_2 = p, x_1 x_2 = q$. Do đó $x_1, x_2 $ đều là các số nguyên dương. Giả sử $x_1 \geq x_2$.
Suy ra $x_2 = 1, x_1 = q$, $1+q = p$. Do đó $p = 3, q=2$.
Thử lại thấy thỏa đề bài.

Bài 25. Tìm tất cả các số nguyên tố $p$ sao cho tổng các ước dương của $p^4$ là một số chính phương.

Lời giải

Theo đề ta có phương trình $1+p+p^2+p^3+p^4 = x^2$.
Ta có $(2p^2+p)^2< 4x^2 < (2p^2+p+2)$.
Do đó $4x^2 = (2p^2+p+1) = 4p^2+4p^3+4p^2+4p+4$
$p^2 -2p – 3 = 0 \Leftrightarrow p=3$.

Bài 26. Tìm tất cả các số nguyên tố $p$ sao cho tồn tại các số nguyên dương $x, y$ thỏa phương trình $x(y^2-p)+y(x^2-p)=5p$.

Lời giải

$(x+y)(xy-p) = 5p$, $x+y \geq 2$ Do đó có các trường hợp sau:\\
$x+y = 5, xy-p=p$. Giải ra được $x=2, y=3, p=3$, $x=3, y=2, p=3$, $x=1, y=4, p=2$, $x=4,y=1, p=2$.\\
$x+y = p, xy -p=5$. $x^2-px+p+5 = 0$. $p^2-4(p+5) = =k^2 \Leftrightarrow (p-2)^2 – 24 = k^2 \Leftrightarrow (p-2-k)(p-2+k) = 24$. \\
Ta có $p-2-k, p-2+k$ cùng chẵn. Có các trường hợp sau:
+ $p-2-k = 2, p-2+k=12$, suy ra $p=9$ (loại)\\
+ $p-2 -k = 4, p-2+k = 6$, suy ra $p=7$. Khi đó $x+y = 7, xy = 12$. Giải ra được $x=3, y=4$ và $x=4, y=3$.

Bài 27. Cho các số nguyên dương $a, b, c, d$ thỏa $ab = cd$. Chứng minh rằng $a + b + c + d$ là hợp số.

Lời giải

Đặt $k = (a,c), a= ka’, c=kc’$, Suy ra $a’b = c’d$, suy ra $b \vdots c’$, đặt $b = mc’$, suy ra $d=ma’$.
Khi đó $a+b+c+d = ka’+mc’ + kc’+ma’ = (k+m)(a’+c’)$ là hợp số.

Bài 28. Tìm tất cả các số nguyên tố $p>q>r$ sao cho $p-r, p-q, q-r$ cũng là các số nguyên tố.

Lời giải

Nếu các số $p, q, r$ đều lẻ, thì $p-r, p-q, q-r$ đề chẵn mà là số nguyên tố và bằng 2, vô lý.
Do đó có 1 số nguyên tố chẳn, suy ra $r = 2$.
$p-2, q-2, p-q$ nguyên tố. Suy ra $p-q = 2$.
Vậy $p-2, p,p+2$ là các số nguyên tố. Suy ra $p-2=3$, $p=5$, $q=7$.

Bài 29. Tìm các số nguyên tố $p,q$ thỏa mãn hệ thức $p + q = {\left( {p – q} \right)^3}$

Lời giải

$p-q = r$ ta có $r^3 =2p+r$. Suy ra $p = \dfrac{r^3-r}{2}$ chia hết cho 3. Suy ra $p=3, q=5$.

Bài 30. Tìm tất cả các số nguyên tố $p$ sao cho hệ phương trình $p+1=2x^2,p^2+ 1=2y^2$ có nghiệm nguyên.

Lời giải

Ta xét $y, x>0$. Ta có $p = 2$ không thỏa.
$p(p-1) = 2(y-x)(y+x)$, suy ra $p |2(y-x)(y+x)$
$p|y-x$, suy ra $2(x+y)|p-1$ (vô lý)
$p|x+y$, mặt khác $p > x, p > y$, suy ra $2p>x+y$, do đó $p = x+y$. Khi đó $p-1 = 2x – 2y$. Từ đó suy ra $x = \dfrac{3p-1}{4}$, thế vào ta giải ra được $p = 7, x = 2, y = 5$.

Bài tập số học ôn thi vào 10 – Phần 2

Bài 11. Chứng minh rằng

a) Trong 5 số nguyên thì có 3 số có tổng chia hết cho 3.
b) Trong 17 số nguyên thì có 9 số có tổng chia hết cho 9.

Giải

a) Một số khi chia cho 3 có các số dư là 0, 1, 2.
Nếu trong 5 số khi chia cho 3 số có đủ 3 số dư 0, 1, 2 thì tổng 3 số này chia hết cho 3.
Nếu có 2 loại số dư thì có 3 số khi chia cho 3 có cùng một số dư, tổng của chúng chia hết cho 3.
Nếu có 1 loại số dư, thì tổng 3 số bất kì đều chia hết cho 3.
b) Đặt các số đó là $a_1, a_2, \cdots, a_{16}, a_{17}$.
Trong 5 số $a_1, \cdots, a_5$ có 3 số có tổng chia hết cho 3, không mất tính tổng quát là $a_1, a_2, a_3$. Đặt $a_1 + a_2 + a_3 = 3b_1$.
Trong 5 số $a_4, \cdots, a_8$ có 3 số có tổng chia hết cho 3, giả sử $a_4, a_5, a_6$ và đặt $a_4 + a_5+ a_6 = 3b_2$.
Tương tự ta xây dựng được các số $b_3, b_4, b_5$.
Khi đó áp dụng tiếp cho 5 số $b_1, b_2, b_3, b_4, b_5$ có 3 số có tổng chia hết cho 3, giả sử $b_1, b_2,b_3$ có tổng chia hết cho 3. Khi đó 9 số $a_1, \cdots, a_9$ có tổng chia hết cho 9.

Bài 12. (Tuyển sinh vào lớp 10 Chuyên Toán trường PTNK 2018)\ Cho $ A_n = 2018^n + 2032^n – 1964^n – 1984^n $ với $ n $ là số tự nhiên.

a) Chứng minh với mọi số tự nhiên $ n $ thì $ A_n $ chia hết cho $ 51 $.
b) Tìm tất cả những số tự nhiên $ n $ sao cho $ A_n $ chia hết cho $ 45. $

Giải

a) \item Do $ 2018 \equiv 1964 \quad \text{(mod 3)} \Rightarrow 2018^n \equiv 1964^n \quad \text{(mod 3)} . $\\
$ 2032 \equiv 1984 \quad \text{(mod 3)} \Rightarrow 2032^n \equiv 1984^n \quad \text{(mod 3)} $.\\
$ \Rightarrow A_n \ \vdots \ 3. $\\
Ta lại có $ 2018 \equiv 1984 \quad \text{(mod 17)} \Rightarrow 2018^n \equiv 1984^n \quad \text{(mod 17)} $.\\
$ 2032 \equiv 1964 \quad \text{(mod 17)} \Rightarrow 2032^n \equiv 1964^n \quad \text{(mod 17)} $.\\
$ \Rightarrow A_n \ \vdots\ 17. $\\
Do $ (3; 17) = 1 $ nên $ A_n \ \vdots \ 51 \quad \forall n$

b) Ta xét các trường hợp của $ n $ để $ A_n \ \vdots \ 5. $
Ta có $ A_n \equiv (-2)^n + 2^n -2\cdot(-1)^n $ (mod 5).
Do đó nếu $ n $ lẻ $ \Rightarrow A_n \equiv 2 \quad $(mod 5)$ \quad \text{(loại)}$.
Nếu $ n = 4k \Rightarrow A_n \equiv 2\cdot 2^{4k} -2 \equiv 2-2 \equiv 0 \quad$ (mod 5) (nhận)
Nếu $ n = 4k + 2 \Rightarrow A_n \equiv 2\cdot 2^{4k+2} -2 \equiv 8 – 2 \equiv 6$ (mod 5) (loại).
Vậy $ A_n \ \vdots \ 5 \Leftrightarrow n \ \vdots \ 4. $
\item Ta xét các trường hợp của $ n $ để $ A_n \ \vdots \ 9. $
Ta có \begin{align*}
A_n &\equiv 2^n + (-2)^n – 2^n – 4^n \quad \text { (mod 9)}\\\\
&\equiv 2^n -4^n \quad \text { (mod 9) \quad (Do n chẵn).} \\\\
& \equiv 2^n(1-2^n) \quad \text { (mod 9)}
\end{align*}
Vì $ (2;9 ) = 1 \Rightarrow 2^n – 1 \ \vdots \ 9$.
Xét $ n= 3k $ với $ k \in \mathbb{N} $. Ta có $ A_n \equiv 2^{3k} – 1 \equiv (-1)^k – 1 \quad \text { (mod 9)} \Rightarrow k$ chẵn
Xét $ n= 3k + 1 $ với $ k \in \mathbb{N} $. Ta có $ A_n \equiv 2^{3k + 1} – 1 \equiv 2\cdot(-1)^k – 1 \quad \text { (mod 9) \quad (loại)}. $
Xét $ n= 3k + 2 $ với $ k \in \mathbb{N} $. Ta có $ A_n \equiv 2^{3k + 2} – 1 \equiv 4\cdot(-1)^k – 1 \quad \text { (mod 9) \quad (loại)}. $

Vậy $ A_n \ \vdots \ 45 \Leftrightarrow n \ \vdots \ 12. $

Bài 13. Tìm các nghiệm nguyên không âm $(x, y)$ của phương trình
${\left( {xy – 1} \right)^2} = {x^2} + {y^2}$

Giải

$(xy-6)^2 – (x+y)^2 = -13$.
$(xy-6-x-y)(xy-6+x+y) = -13$.
Ta có $xy – 6 +x+y \leq xy – 6 -x-y$ nên có các trường hợp.
$xy -6 -x-y = -13, xy -6 +x+y = 1$, giải ra được $(x;y)$ là $(7;0), (0;7)$;
$xy – 6 -x-y=-1, xy-6+x+y = 13$ (VN);
$Vậy phương trình có nghiệm $(0;7), (7;0)$.

Bài 14. Chứng minh rằng phương trình ${y^2} + y = x + {x^2} + {x^3}$ không có nghiệm nguyên dương.

Giải

Ta có $x^3 = (y-x)(y+x+1)$.
Gọi $d$ là ước nguyên tố chung lớn nhất của $y-x, y+x+1$, nếu $d$ là số nguyên tố thì $d|x, d|y$, suy ra $d|1$ (vô lý), Vậy $y-x, y+x+1$ nguyên tố cùng nhau.
Do đó $y -x = a^3, y+x+1 = b^3, ab=x$.
Ta có phương trình $b^3-a^3 = 2ab+1$ với $a, b$ nguyên dương và $b > a\geq 1$. Ta có $b^3-a^3 \geq a^2+b^2+ab > 2ab + 1$.
Vậy phương trình không có nghiệm trong tập các số nguyên dương.

Bài 15. Tìm tất cả các bộ ba số nguyên dương thỏa phương trình:
${\left( {x + y} \right)^2} + 3x + y + 1 = {z^2}$

Giải

Ta có $(x+y)^2 < z^2 < (x+y+2)^2$. Do đó $z^2 = (x+y+1)^2$ hay $(x+y+1)^2 = (x+y)^2+3x+y + 1 \Leftrightarrow y = x$.
\Vậy bộ nghiệm là $(n, n, 2n+1)$ với $n$ là số nguyên dương.

Bài 16. Tìm nghiệm nguyên dương của phương trình sau
$xy + yz + zx – xyz = 2$

Giải

Vai trò của $(x, y, z)$ là như nhau, giả sử $x \geq y \geq z$.
$\dfrac{1}{x}+\dfrac{1}{y}+\dfrac{1}{z} – 1 = \dfrac{2}{xyz} > 0$. Suy ra $\dfrac{3}{z} -1 > 0$, suy ra $z < 3$.
Nếu $z = 1$ thì $x+y = 2$ ta có $x = y = 1$.
Nếu $z=2$ thì $2(x+y)-xy = 2 \Leftrightarrow (x-2)(y-2) = 2$, giải ra được $x = 4, y = 3$.
Do tính đối xứng nên nghiệm của phương trình là $(1, 1, 1), (4,3,2)$ và các hoán vị.

Bài 17. Tìm tất cả các số tự nhiên x, y thỏa: ${5^x} = {y^4} + 4y + 1$

Giải

Có một nghiệm là $(0;0)$.
Dễ thấy $y$ chẵn nên $y^4+4y+1 \equiv 1 (\mod 8)$. Suy ra $x$ chẵn, $x = 2k$. Khi đó $(5^k)^2 = y^4 + 4y+1$ là số chính phương.
Ta có $y\geq 1$ nên $y^4 < y^4+4y + 1 < (y^2+2)^2$. Suy ra $y^4+4y + 1 = (y^2+1)^2 \Leftrightarrow y = 2$, suy ra $x = 2$.
Vậy có 2 cặp nghiệm $(0;0), (2;2)$.

Bài 18. Giải phương trình nghiệm tự nhiên $x – {y^4} = 4$ với $x$ là số nguyên tố.

Giải

$x = y^4+4 = (y^2-2y+2)(y^2+2y+2)$ là số nguyên tố khi và chỉ khi $y^2-2y + 2 = 1$ hay $y=1$. Từ đó $x=1$.

Bài 19. Tìm nghiệm nguyên của phương trình sau
${\left( {{x^2} – {y^2}} \right)^2} = 1 + 16y$

Giải

Dễ thấy nghiệm là $(-1;0), (1;0)$.
Ta có $y \geq 0$, vì $x$ thỏa pt thì $-x$ cũng thỏa nên có thể giả sử $x\geq 0$.
Ta có $(x^2-y^2)^2 = 1 + 16y >1$, suy ra $x^2 > y^2 \Rightarrow x \geq y + 1$.
Nếu $x \geq y + 2$, suy ra $x^2-y^2 \geq 4y + 4 \Rightarrow (x^2-y^2)^2 > 1+16y$.
Do đó $x = y + 1$, suy ra $(1+2y)^2 = 1+16y \Leftrightarrow 4y^2 – 12y = 0 \Leftrightarrow y = 3$. Suy ra $x = 4$.
Vậy nghiệm là $(-4;3), (4;3),(-1;0), (1;0)$.

Bài 20. Chứng minh rằng với mọi số tự nhiên $n > 1$ thì $n^5 + n^4 + 1$ không là số nguyên tố.

Giải

$n^5 + n^4 + 1 = n^5+n^4+n^3-n^3+1 = n^3(n^2+n+1) -(n-1)(n^2+n+1) = (n^2+n+1)(n^3-n+1)$
Mà $n^3-n+1 > 1, n^2+n+1>1$ với mọi $n>1$ nên $n^5+n^4+1$ không là số nguyên tố.

Bài tập số học ôn thi vào lớp 10 chuyên toán – Phần 1

Bài 1. Cho $m, n$ là các số nguyên. Chứng minh rằng nếu $mn+1$ chia hết cho 24 thì $m+n$ cũng chia hết cho 24.

Giải

Ta có $mn+1$ chia hết cho 24, suy ra $mn+1$ chia hết cho 3 và 8. Ta cũng chứng minh $m+n$ chia hết cho 3 và 8.

Nếu $m \equiv p (\mod 3), n \equiv q (\mod 3)$, suy ra $pq + 1 \equiv 0 (\mod 3)$. Suy ra $pq = 2$. Do đó $p = 1, q = 2$ hoặc $p=2, q=1$. Suy ra $p+q \equiv 0 (\mod 3)$ hay $m+n \equiv (\mod 3)$.
Tương tự $m \equiv q (\mod 8), n \equiv p (\mod 8)$, suy ra $pq \equiv 7 (\mod 8)$ và $p, q \in \{1, 2, 3, 4, 5, 6, 7\}$, suy ra $p=1, q=7$ hoặc $p=7, q=1$. Do đó $m+n$ chia hết cho 8.
Vậy $m + n$ chia hết cho 24.5

Bài 2. Tìm tất cả các số $n$ sao cho:

a) $1^n + 2^n + 3^n + 4^n$ chia hết cho 5.
b) $2^{2n} + 2^n + 1$ chia hết cho 21.

Giải

Đặt $A_n = 1^n + 2^n + 3^n + 4^n$.
Nếu $n$ lẻ ta có $1^n + 4^n$ chia hết cho 5, $2^n + 3^n$ chia hết cho 5. Suy ra $1^n + 2^n + 3^n + 4^n$ chia hết cho 5.
Nếu $n$ chẵn, đặt $n = 2k$. Ta có $1 + 2^n + 3^n + 4^n = 1 + 4^k + 9^k + 16^k \equiv 1 + (-1)^k + (-1)^k + 1 (\mod 5)$.
Do đó $A_n \vdots 5 \Leftrightarrow k$ lẻ.
Vậy $A_n$ chia hết cho 5 khi và chỉ khi $n$ lẻ hoặc $n$ chia 4 dư 2.

Đặt $B_n = 2^{2n} + 2^n + 1$.
Ta tìm $n$ để $B_n$ chia hết cho 3 và 7.

Nếu $n = 2k$ ta có $B_n = 16^k + 4^k + 1 \equiv 0 (\mod 3)$.\\
Nếu $n = 2k + 1$ ta có $B_n = 4\cdot 16^k + 2\cdot 4^k + 1 \equiv 7 (\mod 3)$ (loại)\\
Vậy $B_n \vdots 3 \Leftrightarrow n = 2k$.

Nếu $n = 3k$ ta có $B_n = 64^k + 8^k + 1 \equiv 3 (\mod 7)$. (loại)\\
Nếu $n = 3k+ 1$ ta có $B_n = 4 \cdot 64^k + 2 \cdot 8^k + 1 \equiv 0 (\mod 7)$ (nhận)
Nếu $n = 3k + 2 $ ta có $B_n = 16\cdot 64^k + 4\cdot 8^k + 1 \equiv 0 (\mod 7)$.

Vậy $B_n$ chia hết cho 7 khi và chỉ khi $n = 6k+4$ hoặc $n = 6k+2$.

Bài 3. (Tuyển sinh vào lớp 10 Chuyên Toán trường PTNK 1997)

a) Tìm tất cả các số nguyên dương $n$ sao cho $n2^n + 3^n$ chia hết cho 5.
b) Tìm tất cả các số nguyên dương $n$ sao cho $n2^n + 3^n $ chia hết cho 25.

Giải

a) Nếu $n$ ta có $2^n + 3^n$ chia hết cho 5.
Xét $n=2k + 1$ ta có $n.2^n + 3^n = (n-1)2^n+ 2^n + 3^n$ chia hết cho 5 khi và chỉ khi $n-1$ chia hết cho 5, hay $k$ chia hết cho 5,suy ra $k = 5q$. Vậy $n = 10q + 1$.
Xét $n = 2k$ ta có $n.2^n + 3^n = 2k.4^k + 9^k = 2k.4^k + 4^k + 9^k – 4^k = (2k+1).4^k + 9^k – 4^k $ chia hết cho 5 khi $2k+1$ chia hết cho 5. Khi đó $k = 5q + 2$, suy ra $n = 10q + 4$.
Vậy với $n = 10q + 1, 10q + 4$ thì $n.2^n + 3^n$ chia hết cho 5.

b) Theo câu a để $A=n.2^n + 3^n$ chia hết cho 5 thì $n = 10q+1, 10q + 4$. Ta tìm $q$ để $n.2^n + 3^n$ chia hết cho 25.
+Với $n = 10q + 1$ ta có $A = (10q+1)2^{10q+1} + 3^{10q+1} = (20q+2).1024^q + 3.3^{10q}$\\
Ta có $1024 \equiv -1 (\mod 25), 3^10 \equiv -1 (\mod 25)$. Suy ra $A \equiv (20q + 2)(-1)^q + 3.(-1)^q (\mod 25)$ hay $A = (-)^q (20q+5) (\mod 25)$.
Suy ra $A$ chia hết cho 25 khi và chỉ khi $20q +5$ chia hết cho 25 hay $4q+1$ chia hết cho 5. Suy ra $q = 5k + 1$. Vậy $n = 10(5k+1)+1 = 50k + 11$.
+Với $n = 10q + 4$. Ta có $A = (10q+4)2^{10q+4} + 3^{10q+4} = (160q+64)2^{10q} + 81.3^{10q} \equiv (10q+14)(-1)^q + 6(-1)^q (\mod 25) \equiv (-1)^q(10q+20) (\mod 25)$.
Do đó $A$ chia hết cho 25 khi và chỉ khi $10q+20$ chia hết cho 25 hay $q+2$ chia hết cho 5, suy ra $q = 5k + 3$. Suy ra $n = 10(5k+3) + 4 = 50k + 34$.
Vậy $n = 50k+11, 50k+34$.

Bài 4. (Tuyển sinh vào lớp 10 Chuyên Toán trường PTNK 1997)

a) Tìm tất cả các số nguyên dương sao cho $2^n – 1$ chia hết 7.
b) Cho số nguyên tố $p \geq 5$. Đặt $A = 3^p – 2^p – 1$. Chứng minh $A$ chia hết cho $42p$.

Giải

a)
TH1: $n = 3k$ ta có $2^n – 1 = 2^{3k}-1 = 8^k – 1$ chia hết cho 7.
TH2: $n = 3k + 1$ ta có $2^n- 1= 2.8^k – 1$ chia 7 dư 1.
TH3: $n = 3k + 2$ ta có $2^n – 1= 4.8^k – 1$ chia 7 dư 3.
Vậy $2^n- 1$ chia hết cho 7 khi và chỉ khi $n$ chia hết cho 3.

b)
$42p = 2.3.7.p$.
TH1: $p = 7$ ta có $3^7 – 2^7 – 1$ chia hết cho $42.7$.
TH2: $p > 7$ khi đó các số $2, 3, 7, p$ đôi một nguyên tố cùng nhau.
+ Ta có $3^p – 1 – 2^p$ chia hết cho 2.
+ $2^p + 1$ chia hết cho 3 vì $p$ lẻ, suy ra $3^p -2^p-1$ chia hết cho 3.
+ $p$ nguyên tố lớn hơn hoặc bằng 5, suy ra $p = 6k + 1$ hoặc $p = 6k+5$. Nếu $p = 6k + 1$ ta có $3^p – 2^p – 1 = 3^{6k+1} – 2^{6k+1} – 1 = 3.3^{6k} – 2.2^{6k} – 1$.
Ta có $3^6 \equiv 1 (\mod 7)$, suy ra $3^{6k} \equiv 1 (\mod 7)$, tương tự thì $2^{6k} \equiv 1 (\mod 7)$. Do đó $3.3^{6k} – 2.2^{6k} – 1 \equiv 0 (\mod 7)$.
Nếu $p = 6k + 5$ ta có $3^p – 2^p – 1 \equiv 3^5 – 2^5 – 1 \equiv 0 (\mod 7)$.
Do đó $3^p – 2^p – 1$ chia hết cho 7.
+ Theo định lý Fermat nhỏ, ta có $3^p \equiv 3 (\mod p), 2^p \equiv 2 (\mod 7)$. Suy ra $3^p – 2^p – 1$ chia hết cho $p$.
Vậy $3^p – 2^p – 1$ chia hết cho $42p$.

Bài 5. Cho a,b là hai số nguyên dương thỏa mãn $4{a^2} – 1$ chia hết cho $4ab – 1$. Chứng minh rằng $a = b$.

Giải

$4a^2-1$ chia hết cho $4ab-1$ suy ra $4a^2\geq 4ab \Rightarrow a\geq b$.
Ta có $4a^2 – 1 \vdots 4ab-1 \Rightarrow 4b^2(4a^2-1) \vdots 4ab – 1 \Rightarrow 16a^2b^2-1-(4b^2-1) \vdots 4ab-1$, suy ra $4b^2-1 \vdots 4ab-1$. Tương tự trên ta có $b \geq a$.
Do đó $a = b$.

Bài 6. Cho các số nguyên $x, y, z$ thỏa $(x-y)(y-z)(z-x) = x+ y + z$. Chứng minh rằng $x + y + z$ chia hết cho 27.

Giải

Nếu $x, y, z$ khi chia cho 3 có số dư khác nhau thì $x+y+z \vdots 3$ nhưng $(x-y)(y-z)(z-x)$ không chia hết cho 3 (mẫu thuẫn).
Nếu 2 trong 3 số $x, y,z$ có số dư giống nhau, giả sử là $x, y$. Khi đó $x-y \vdots 3$, suy ra $(x-y)(y-z)(z-x)$ chia hết cho 3, nhưng $x+y + z$ không chia hết cho 3 (mâu thuẫn).
Vậy $x, y, z$ có cùng số dư khi chia cho 3, suy ra $x-y, y-z, z-x$ đều chia hết cho 3. Do đó $x+y+z = (x-y)(y-z)(z-x)$ chia hết cho 27.

Bài 7. Cho $a_n = 2^{2n+1} + 2^{n+1} + 1$ và $b_n = 2^{2n+1} – 2^{n+1} + 1$. Chứng minh rằng với mỗi số tự nhiên $n$, có một và chỉ một trong hai số $a_n, b_n$ chia hết cho 5.

Giải

$a_nb^n = (2^{2n+1}-2^{n+1}+1)(2^{2n+1}+2^{n+1}+1) = (2^{2n+1}+1)^2 – (2^{n+1})^2 = 4^{2n+1} +2.2^{2n+1} + 1 – 2^{2n+2} = 4^{2n+1} + 1$.
Ta có $4 \equiv -1(\mod 5)$, suy ra $4^{2n+1} \equiv -1 (\mod 5)$. Suy ra $4^{2n+1} + 1 \equiv 0(\mod 5)$.
Vậy $a_nb_n$ chia hết cho 5 với mọi $n$.
Ta có $a_n + b_n = 2.2^{2n+1} + 2 = 4^{n+1} + 2$.
Ta có $4^{n+1} \equiv -1, 1 (\mod 5)$. Suy ra $4^{n+1} +2 \equiv 1, 3 (\mod 5)$. Vậy $a_n + b_n$ không chia hết cho 5 với mọi $n$.
Do đó chỉ có một trong 2 số $a_n, b_n$ chia hết cho 5.

Bài 8. Cho $n$ là số tự nhiên. Chứng minh rằng $3^nn^3+1$ chia hết cho 7 khi và chỉ khi $3^n + n^3$ chia hết cho 7.

Giải

Nếu $3^nn^3 + 1$ chia hết cho 7. Suy ra $n$ không chia hết cho 7, suy ra $n^6-1$ chia hết cho 7.\\
Ta có $n^3 (3^n + n^3 ) = n^33^n + n^6 = n^33^n +1 + n^6 – 1$ chia hết cho 7. \\
Mà $(n,7) = 1$. Suy ra $3^n + n^3$ chia hết cho 7.
Nếu $3^n + n^3$ chia hết cho 7. Làm tương tự ta cũng có $n^33^n + 1$ chia hết cho 7.

Bài 9. Chứng minh rằng nếu $2^n-1$ là số nguyên tố thì $n$ cũng là số nguyên tố.

Giải

Giả sử $n$ không là số nguyên tố.
Nếu $n = 1$ thì $2^1 – 1$ không nguyên tố.
Nếu $n$ là hợp số, ta có $n = pq$ với $1 < p < n$.
Khi đó $2^n – 1= (2^p)^q -1$ chia hết cho $2^p-1$. Mà $1< 2^p-1 < 2^n-1$ nên $2^n-1$ không là số nguyên tố. (Vô lý).

Bài 10. Ta điền các số từ 1 đến 9 vào bảng vuông $3\times 3$ sao cho mỗi số điền một lần, tổng các số cùng một hàng, một cột và đường chéo chia hết cho 9. Chứng minh rằng ô chính giữa bảng luôn là một số chia hết cho 3.

Giải

Giả sử các số là $a, b, c, d, e, f, g, h,i$ trong đó $e$ là ô chính giữa.

a  b  c
d  e  f
g  h  i

Ta có $a+e+i + d+e+f + c+e+g + b+e+h = (a+b+c+d+e+f+g+h+i) +3e \vdots 9$, mà $a +b+c+d+e+f+g+h+i = 1+2+\cdots + 9 = 45$ chia hết cho 9.
Suy ra $3e\, \, \vdots 9$, do đó $e \,\vdots \, 3$.

Bài tập số chính phương – Lớp 9

Bài 1. Chứng minh rằng

a) Một số chính phương chia 3 dư 0 hoặc 1.
b) Một số chính phương chia 4 dư 0 hoặc 1.
c) Một số chính phương chia 5 dư 0, 1 hoặc 4.
Bài 2. Chứng minh rằng một số là số chính phương khi và chỉ khi số ước của số đó là một số lẻ.

Bài 3. Chứng minh rằng nếu tổng hai số chính phương chia hết cho 3 thì tích của nó sẽ chia hết 81.

Bài 4. Chứng minh rằng với $n$ là số tự nhiên thì $3n-1, 5n + 2, 5n – 2, 7n-2, 7n+3$ không phải là số chính phương.

Bài 5. Tìm tất cả các số tự nhiên $n$ sao cho $n.2^{n+1}+1$ là một số chính phương.

Bài 6. Chứng minh rằng nếu $x^2+ 2y$ là một số chính phương với $x, y$ nguyên dương thì $x^2+ y$ là tổng của hai số chính phương.

Bài 7. Chứng minh rằng nếu $3x + 4y,3y + 4x$ là các số chính phương thì $x,y$ đều chia hết cho 7.

Bài 8. Cho các số nguyên dương $a, b$. Giả sử các số $a + 2b,b + 2a$ đều là bình phương của một số nguyên thì $a$ và $b$ đều chia hết cho 3.

Bài 9. Cho các số tự nhiên $a, b, c$ thỏa: $a + 2b,b + 2c,c + 2a$ đều là bình phương của một số tự nhiên.
a)Chỉ ra một bộ số thỏa đề bài.
b) Giả sử trong 3 số $a + 2b,b + 2c,c + 2a$ có một số chia hết cho 3. Chứng minh rằng: $P = \left( {a – b} \right)\left( {b – c} \right)\left( {c – a} \right)$ chia hết cho 27.

Bài 10. Chứng minh rằng nếu $\overline {abc} $ là một số nguyên tố thì ${b^2} – 4ac$ không phải là một số chính phương.

Bài 11. Tìm tất cả các số tự nhiên $n \geq 2$ sao cho tồn tại $n$ số nguyên liên tiếp mà tổng của chúng là một số chính phương.

Bài 12. Tìm $d$ sao cho với mọi $a,b \in {2,5,d}$ thì $ab-1$ là một số chính phương.

Bài 13. Chứng minh rằng với mọi $d$ thì tập ${2,5,13,d}$ luôn tồn tại hai số $a,b \in {2,5,13,d}$ sao cho $ab-1$ không phải là số chính phương.

Bài 14. Chứng minh rằng nếu tích của hai số nguyên tố cùng nhau là một số chính phương thì mỗi số cũng là số chính phương.

Bài 15. Cho các số nguyên dương $a, b$ thỏa $2{a^2} + a = 3{b^2} + b$.

a)Tìm $a, b$ biết $a$ và $b$ là hai số nguyên tố cùng nhau.
b) Chứng minh $a-b$ và $2a + 2b + 1$ là các số chính phương.

Bài 16. Cho các số nguyên $a, b, c$ thỏa $a + b + c$ chia hết cho 6 và ${a^2} + {b^2} + {c^2}$ chia hết cho 36. Đặt $A = {a^3} + {b^3} + {c^3}$

a) Chứng minh rằng A chia hết cho 8.
b) A có chia hết cho 27 không? Tại sao?

Bài 17. Cho $a,b,c$ là ba số nguyên dương thỏa $\dfrac{1}{a} – \dfrac{1}{b} = \dfrac{1}{c}$. Gọi $d$ là ước chung lớn nhất của ba số đó . Chứng minh rằng $d(b – a)$ là số chính phương.

 

Bài 18. Tìm tất cả các số nguyên dương $n$ sao cho $T = {2^n} + {3^n} + {4^n}$ là số chính phương.

 

Bài 19. Tìm tất cả các cặp số nguyên $a, b$ sao cho $3^a+ 7^b$ là một số chính phương.

Bài 20. (Chuyên Thái Bình 2021) Giả sử $n$ là số tự nhiên thỏa mãn điều kiện $n(n+1)+7$ không chia hết cho 7. Chứng minh rằng $4 n^{3}-5 n-1$ không là số chính phương.

Bài  21 (Thanh Hóa – Chuyên Tin 2021) Cho số tự nhiên $n \geqslant 2$ và số nguyên tố $p$ thỏa mãn $p-1$ chia hết cho $n$ và $n^{3}-1$ chia hết cho $p$. Chứng minh rằng $n+p$ là một số chính phương.

Bài 22 (Chuyên Lê Khiết) Cho các số nguyên tố $p, q$ thỏa mãn $p+q^{2}$ là số chính phương. Chứng minh rằng
a) $p=2 q+1$.
b) $p^{2}+q^{2021}$ không phải là số chính phương.

Bài 23 (Kiên Giang 2021) Cho $m, p, r$ là các số nguyên tố thỏa mãn $m p+1=r$. Chứng minh rằng $m^{2}+r$ hoặc $p^{2}+r$ là số chính phương.

Bài 24. (Chuyên Tiền Giang) Cho $m, n$ là các số nguyên dương sao cho $m^{2}+n^{2}+m$ chia hết cho $m n$. Chứng minh rằng $m$ là số chính phương.

Bài 25.(Chuyên Phổ thông Năng khiếu – ĐHQG thành phố Hồ Chí Minh 2021-2022)

a) Tìm tất cả số tự nhiên $n$ sao cho $(2 n+1)^{3}+1$ chia hết cho $2^{2021}$.
b) Cho số tự nhiên $n$ và số nguyên tố $p$ sao cho $a=\frac{2 n+2}{p}$ và $b=\frac{4 n^{2}+2 n+1}{p}$ là các số nguyên. Chứng minh rằng $a$ và $b$ không đồng thời là các số chính phương.

 

 

Số hữu tỉ – Vô tỉ

Số hữu tỉ – Số vô tỉ

(Bài viết dành cho các em trung học cơ sở)

Trong bài viết nhỏ này tôi xin giới thiệu một số bài toán liên quan đến các tập hợp số hữu tỉ và vô tỉ, một số trong đó đã xuất hiện trong các kì thi tuyển sinh vào 10 hay các kì thi học sinh giỏi.
Đầu tiên ta xem lại một số khái niệm và tính chất quan trọng.

Định nghĩa. Tập hợp các số có dạng $\dfrac{p}{q}$ trong đó $p, q$ là các số nguyên, $q \neq 0$ được gọi là số hữu tỉ. Kí hiệu là $\mathbb{Q}$. Tập số nguyên là tập con của tập các số hữu tỉ.
Tập hợp các số không phải là số vô tỉ được gọi là số vô tỉ, kí hiệu là $I$.

Tính chất 1. Ta có một số tính chất sau của số vô tỉ và hữu tỉ.

  • Tổng hiệu tích thương của hai số hữu tỉ là hữu tỉ.
  • Tổng, tích, thương của một số hữu tỉ và vô tỉ là một số vô tỉ

Việc chứng minh một số là số hữu tỉ hay vô tỉ chủ yếu dựa vào các định nghĩa trên, trong đó việc chứng minh một số là số vô tỉ hầu hết là sử dụng phương pháp chứng minh phản chứng.
Ta bắt đầu với bài toán cơ bản sau:
Ví dụ 1.
a) Chứng minh $\sqrt{2}$ là một số vô tỉ.
b) Chứng minh $\sqrt{2}+\sqrt{3}$ là một số vô tỉ.

Lời giải.

Ta sử dụng phương pháp chứng minh là phản chứng.

a) Giả sử $\sqrt{2}$ là số hữu tỉ, tức là tồn tại $\dfrac{p}{q}$ trong đó $p, q \in \mathbb{Z},(p,q) = 1, q \neq 0$ và $\sqrt{2}=\dfrac{p}{q}$.
Khi đó ta có $p^2 = 2q^2$, suy ra $p^2$ chia hết cho $2$ mà $2$ nguyên tố nên $p$ chia hết cho $2$, $p = 2k$.
Suy ra $q^2 = 2k^2$, lí luận tương tự thì $q$ chia hết cho $2$, do đó $(p, q) \neq 1$ (mâu thuẫn).
Vậy điều giả sử sai, $\sqrt{2}$ là số vô tỉ.
b) Giả sử $\sqrt{2}+\sqrt{3} = a$ hữu tỉ, suy ra $\sqrt{6} = \dfrac{a^2-5}{2}$ hữu tỉ. Chứng minh tương tự trên ta cũng suy ra điều vô lí.

Từ bài toán trên ta có thể chứng minh bài toán tổng quát sau:

Ví dụ 2. Cho $n$ là số tự nhiên nếu $\sqrt{n}$ không là số tự nhiên thì $\sqrt{n}$ là số vô tỉ.

Lời giải.

Giả sử $\sqrt{n}$ không phải vô tỉ và không phải số nguyên, suy ra $\sqrt{n} = \dfrac{p}{q}$ trong đó $(p,q) =1, q > 1$.
Tương tự ta có $p^2 = nq^2$. Do $q > 1$ nên có ước nguyên tố, giả sử $r$ là một ước nguyên tố của $q$, suy ra $p^2$ chia hết cho $r$, suy ra $p$ chia hết cho $r$, khi đó $(p,q) \neq 1$ (vô lí).
Vậy căn của một số nguyên là một số nguyên hoặc là một số vô tỉ.
\

Đặt $\sqrt{2} = x$, ta có $x^2 = 2 \Leftrightarrow x^2 – 2 = 0$, đến đây ta thấy $\sqrt{2}$ là một nghiệm của phương trình $x^2-2 = 0$. Ta có thể chứng minh phương trình $x^2 -2=0$ không có nghiệm hữu tỉ, từ đó suy ra $\sqrt{2}$ không là số hữu tỉ. Tất nhiên việc chứng minh này không khác mấy chứng minh trên. Tuy nhiên với các nhìn khác, ta có bài toán sau:

Ví dụ 3. Cho phương trình với các hệ số nguyên $a_0, a_1, \cdots, a_n$: $$a_nx^n + a_{n-1}x^{n-1}+\cdots+a_1x + a_0 = 0$$
Khi đó nếu $\dfrac{p}{q}$ với $(p,q)=1$ là một nghiệm hữu tỉ của phương trình thì $p|a_0, q|a_n$.Đặt biệt nếu $a_n=1$ thì nếu phương trình có nghiệm hữu tỉ thì nghiệm là số nguyên.

Lời giải

Thế $\dfrac{p}{q}$ vào phương trình và qui đồng, ta có $$a_np^n+a_{n-1}qp^{n-1}+\cdots+a_1q^{n-1}p + a_0q^n = 0$$
Khi đó $a_np^n$ chia hết cho $q$, suy ra $a_n$ chia hết cho $q$, tương tự thì $a_0$ chia hết cho $p$.

Cũng tương tự, ta có bài toán sau:
Ví dụ 4. Cho phương trình $ax^2 + bx + c = 0$, trong đó $a, b, c$ là các số tự nhiên lẻ. Chứng minh rằng phương trình không có nghiệm hữu tỉ.
Lời giải.

Giả sử $\dfrac{p}{q}, (p,q)=1$ là một nghiệm hữu tỉ của phương trình trên. Khi đó ta có $p|c, q|a$, suy ra $p, q$ đều lẻ. Mặt khác ta có $ap^2 + bpq+ cq^2 = 0$. Vế trái là một số lẻ nên vô lí. Vậy phương trình không có nghiệm hữu tỉ.

Sử dụng bài toàn 3 ta có thể chứng minh $\sqrt{2} + \sqrt{6}$ là số vô tỉ theo một các khác. Bằng cách chứng minh $a = \sqrt{2}+\sqrt{6}$ là nghiệm của phương trình bậc 4: $x^4 – 10x^2 – 1 = 0$, và dễ thấy phương trình trên không có nghiệm hữu tỉ nên $\sqrt{2}+\sqrt{6}$ là số vô tỉ.

Sau đây ta đi tới một số bài toán khác cũng liên quan đến số hữu tỉ và vô tỉ.
Ví dụ 5. Cho các số thực $x, y, z$ khác 0 thỏa $xy, yz, xz$ là các số hữu tỉ.
a) Chứng minh $x^2 + y^2 + z^2 $ là số hữu tỉ.
b) Giả sử $x^3+y^3+z^3$ cũng là số hữu tỉ. Chứng minh $x, y, z$ là các số hữu tỉ.
Lời giải.

a) Ta có $xy, yz \in \mathbb{Q}$, suy ra $\dfrac{x}{z} \in \mathbb{Q}$.
Mà $xz \in \mathbb{Q}$ suy ra $x^2 \in \mathbb{Q}$.
Tương tự ta cũng có $y^2, z^2 \in \mathbb{Q}$.
b) Ta có $x(x^3+y^3+z^3) = (x^2)^2 + (xy)y^2 + (xz)z^2 \in \mathbb{Q}$. Suy ra $x \in \mathbb{Q}$.
Tương tự ta cũng có $y, z \in \mathbb{Q}$.

Chú ý. Với cách giải trên ta chấp nhận không thể xảy ra $x^3+y^3+z^3 = 0$ vì phương trình này không có nghiệm nguyên hay nghiệm hữu tỷ.

Ví dụ 6. Tìm tất cả các số tự nhiên $a, b$ sao cho $$\dfrac{\sqrt{2}+\sqrt{a}}{\sqrt{3}+\sqrt{b}}
$$ là số hữu tỉ.
Lời giải.
Đặt $x = \dfrac{\sqrt{2}+\sqrt{a}}{\sqrt{3}+\sqrt{b}}$ là số nguyên.
Suy ra $\sqrt{a} – x\sqrt{b} = x\sqrt{3}-\sqrt{2}$
Bình phương hai vế ta có $a +x^2b -2x\sqrt{ab} = 3x^2+2-2x\sqrt{6} \Rightarrow a+x^2b-3x^2-2 = 2x(\sqrt{ab}-\sqrt{6})$.
Suy ra $\sqrt{ab}-\sqrt{6} = y \in \mathbb{Q}$.
Khi đó $ab = 6+y^2 – 2y\sqrt{6}$.
Vì $\sqrt{6}$ là số vô tỉ nên đẳng thức xảy ra khi và chỉ khi $y = 0$ và $ab=6$.
Ta xét các trường hợp sau:

  • $a = 1, b = 6 \Rightarrow x = \dfrac{1}{\sqrt{6}}$ vô tỉ.
  • $a = 2, b = 3 \Rightarrow x= \dfrac{\sqrt{2}}{\sqrt{3}}$.
  • $a = 3, b = 2 \Rightarrow x = 1$.
  • $a = 6, b = 1 \Rightarrow x = \sqrt{2}$ vô tỉ.

Vậy $a = 3, b = 2$ là số cần tìm.

Ví dụ 7. Tìm tất cả các bộ số hữu tỉ dương $(x, y, z)$ sao cho $x+\dfrac{1}{y}, y + \dfrac{1}{z}, z+\dfrac{1}{x}$ là các số nguyên.

Lời giải.
Đặt $a = x+\dfrac{1}{y} (1), b = y + \dfrac{1}{z} (2), c = z+\dfrac{1}{x} (3)$.
Từ (1) ta có $y = \dfrac{1}{a-x}, z = \dfrac{1}{b-y} = \dfrac{a-x}{ab-1-bx}$. Thế vào (3) ta có:
$\dfrac{a-x}{ab-1-bx}+\dfrac{1}{x} = c \Leftrightarrow (bc-1)x + (a-b+c-abc)x + ab – 1 = 0$ (4).
Nếu $bc = 1$ thì $b = 1, c = 1$ suy ra $a = 1$. Khi đó $3 = x + \dfrac{1}{x} + y +\dfrac{1}{y} + z + \dfrac{1}{z} \geq 6$ (vô lý)
Nếu $bc \neq 1$, khi đó ta xem (4) như phương trình bậc hai có nghiệm hữu tỷ $x$, khi đó $\Delta = (a-b+c-abc)^2 – 4(bc-1)(ab-1) = (abc-a-b-c)^2 – 4$ là số chính phương.
Đặt $t = abc-a-b-c$ ta có $t^2-4 = k^2$, giải ra được $ t = 2$ hoặc $t = -2$.

$0=abc-a-b-c +2 = a(bc-1) – b-c+2 \geq bc-b-c+1 = (b-1)(c-1)$. Suy ra $b = c=1$ (vô lý).
$0=abc-a-b-c-2 \geq (b-1)(c-1) – 4\Rightarrow (b-1)(c-1) \leq 4$.
Nếu $(b-1)(c-1) = 4$ thì $b = 2, c=5$; $b = 3, c=3$; $b=5, c=2$. Trong các trường hợp này thì $a=1$.
Nếu $ a= 1, b = 2, c = 5$ giải được $(x, y, z) = (\dfrac{1}{3}, \dfrac{3}{2},2)$.
Nếu $a = 1, b = 3, c = 3$ thì $(x, y, z) = (\dfrac{1}{2},2,1)$.
Nếu $a = 1, b = 5, c = 2$ thì $(x, y, z) = (\dfrac{2}{3}, 3,2)$.
Nếu $(b-1)(c-1) = 3 \Rightarrow bc= b+c +2 = abc-a = a(bc-1) \Rightarrow bc-1|bc \Rightarrow bc = 1, a = 1$. (loại)
Khi $(b-1)(c-1) =2 \Rightarrow a = b = c = 2$, giải ra được $(x, y, z) = (1, 1, 1)$.

Trên đây là một số bài toán liên quan đến số hữu tỉ, vô tỉ, hi vọng các em có thêm kinh nghiệm để làm bài trong các tình huống này. Sau đây là một số bài tập rèn luyện.

Bài 1.  Tìm một đa thức hệ số nguyên nhận $\alpha = 2 + \sqrt[3]{2} + \sqrt[3]{4}$ làm nghiệm. Chứng minh $\alpha$ là số vô tỷ.
Bài 2.  Cho các số $a, b$ sao cho $a – \sqrt{ab}$ và $b-\sqrt{ab}$ đều là các số hữu tỉ. Chứng minh rằng $a, b$ cũng là các số hữu tỉ.
Bài 3. Ta nói các căp số $(\mathrm{a}, \mathrm{b}) a \neq b$, là có tính chất $\mathrm{P}$ nếu $a^{2}+b \in Q$ và $b^{2}+a \in \mathbb{Q}$.
Chứng minh rằng:
a) Các số $a=\dfrac{1+\sqrt{2}}{2}, b=\dfrac{1-\sqrt{2}}{2}$ là các số yô tỷ có tính chất $\mathrm{P}$.
b) Nếu $(\mathrm{a}, \mathrm{b})$ có tính chất $\mathrm{P}$ và $a+b \in \mathbb{Q} \backslash{1}$ thì $a, b$ à các số hũu tỷ.
c) Nếu $(\mathrm{a}, \mathrm{b})$ có tính chất $\mathrm{P}$ và $\dfrac{a}{b} \in \mathbb{Q}$ thì $\mathrm{a}, \mathrm{b}$ là các số hũu tỷ.
Bài 4.  Với mỗi số hữu tỷ $q$ đặt $V_q = {x \in \mathbb{Q}|x^3-2015x =q}$.

a)Tìm $q$ sao cho $V_q$ có là tập rỗng và $V_q$ có đúng một phần tử.
b) Gọi $S(V_q)$ là số phần tử của $V_q$, tìm tất cả các giá trị của $S(V_q)$.
Bài 5.
a) Cho số thực $x$ thỏa $x^2+x$ và $x^3+2x$ là số hữu tỷ. Chứng minh $x$ cũng là số hữu tỷ.
b) Chứng minh rằng tồn tại số vô tỷ $x$ sao cho $x^2+x$ và $x^3-2x$ là hữu tỷ.

Hết

Bội chung – Bội chung nhỏ nhất

Bội chung. Một số là bội chung của hai hay nhiều số khi nó là bội của tất cả các số đó.

Kí hiệu bội chung của $a, b$ là BC(a, b).

Ví dụ 1. B(4) = {0, 4, 8, 12, 16, 20,…} và B(6) = {0, 6, 12, 18, 24, 30,…}

Thì BC(4,6) = {0, 12, 24, …}

Cách tìm bội chung của a và b

  • Tìm tập các bội của a là B(a), tìm bội của b là B(b)
  • Tìm các phần tử của của B(a) và B(b), ta được BC(a, b).

Bội chung nhỏ nhất. 

Bôi chung nhỏ nhất của hai hay nhiều số là số khác 0 nhỏ nhất trong tập các bội chung của nó.

Kí hiệu là BCNN(a,b).

Chú ý. Nếu $a \neq 1$ thì BCNN(a,1) = a và BCNN(a,b,1) = BCNN(a,b).

Ví dụ 2. Một lớp có không quá 42 học sinh. Nếu xếp hàng 4 hoặc hàng 6 thì vừa đủ. Nếu xếp hàng 5 thì thừa 1 em. Hỏi lớp đó có bao nhiêu học sinh?
Lời giải.
Số học sinh của lớp đó là bội chung của 4 và 6 .

Ta có $\mathrm{BCNN}(4,6)=12$ nên $\mathrm{BC}(4,6)={0 ; 12 ; 24 ; 36 ; 48 ; \ldots}$.
Vi số học sinh của lớp đó không quá 42 và là một số chia cho 5 dư 1 nên lớp đó có 36 học sinh.

Tìm bội chung nhỏ nhất bằng cách phân tích các số ra thừa số nguyên tố

Muốn tìm BCNN của hai hay nhiều số lớn hơn 1 , ta thực hiện ba bước sau:

  • Bước 1: Phân tích mỗi số ra thừa số nguyên tố.
  • Bước 2: Chọn ra các thừa số nguyên tố chung và riêng.
  • Bước 3: Lập tích các thừa số đã chọn, mỗi thừa số lấy với số mũ lớn nhất của nó. Tích đó là BCNN phải tìm.

Ví du 5: Tìm BCNN của 12,90 và 150 .
Lời giải.
– Phân tích mỗi số $12,90,150$ ra thừa số nguyên tố:
$$
12=2^{2} \cdot 3 ; 90=2 \cdot 3^{2} \cdot 5 ; 150=2 \cdot 3 \cdot 5^{2} .
$$
– Các thừa số nguyên tố chung và riêng là 2,3 và 5 .
– Lập tích các thừa số chung và riêng đã chọn ở trên, mỗi thừa số lấy với số mũ lớn nhất của nó: $2^{2} \cdot 3^{2} \cdot 5^{2}$
Vậy $\operatorname{BCNN}(12,90,150)=2^{2} \cdot 3^{2} \cdot 5^{2}=900$.

Ứng dụng trong quy đổng mẫu các phân số

Muốn quy đồng mẫu số nhiều phân số ta có thể làm như sau:

  • Bước 1: Tìm một bội chung của các mẫu số (thường là BCNN) để làm mẫu số chung.
  • Bước 2: Tìm thừa số phụ của mỗi mẫu số (bằng cách chia mẫu số chung cho từng mẫu số riêng).
  • Bước 3: Nhân tử số và mẫu số của mỗi phân số với thừa số phụ tương ứng.

Ví dụ 6. Ta có thể quy đồng mẫu hai phân số $\frac{1}{6}$ và $\frac{5}{8}$ theo hai cách như sau:
Ta có: 48 là một bội chung của 6 và 8 ; Ta có: $\mathrm{BCNN}(6,8)=24$;

Do đó: $\quad 24: 6=4 ; 24: 8=3$.

$\frac{1}{6}=\frac{1.4}{6.4}=\frac{4}{24}$ và $\frac{5}{8}=\frac{5.3}{8.3}=\frac{15}{24}$.

 

Bài tập rèn luyện.

Bài 1. Tìm:
a) $\mathrm{BC}(6,14)$;
b) $\mathrm{BC}(6,20,30)$
c) $\mathrm{BCNN}(1,6)$
d) $\mathrm{BCNN}(10,1,12)$;
e) $\mathrm{BCNN}(5,14)$.
Bài 2. a) Ta có $\mathrm{BCNN}(12,16)=48$. Hãy viết tập hợp A các bội của 48 . Nhận xét về tập hợp $\mathrm{BC}(12,16)$ và tập hợp $\mathrm{A}$.
b) Để tìm tập hợp bội chung của hai số tự nhiên a và b, ta có thể tìm tập hợp các bội của $\mathrm{BCNN}(\mathrm{a}, \mathrm{b})$. Hãy vận dụng để tìm tập hợp các bội chung của:
i. 24 và 30 ; $\quad$ ii. 42 và 60 ; $\quad$ iii. 60 và 150 ; $\quad$ iv. 28 và 35 .
Bài 3. Quy đồng mẫu số các phân số sau (có sử dụng bội chung nhỏ nhất):
a) $\frac{3}{16}$ và $\frac{5}{24}$;
b) $\frac{3}{20} ; \frac{11}{30}$ và $\frac{7}{15}$

Bài 4. Chị Hoà có một số bông sen. Nếu chị bó thành các bó gồm 3 bông, 5 bông hay 7 bông thì đều vừa hết. Hỏi chị Hoà có bao nhiêu bông sen? Biết rằng chị Hoà có khoảng từ 200 đến 300 bông.

Ước chung lớn nhất

Ước chung

  • Một số được gọi là ước chung của hai hay nhiều số nếu nó là ước của tất cả các số đó.
  • Tập các ước chung của $a$ và $b$ kí hiệu ƯC(a,b). Ta có x thuộc ƯC(a,b) khi và chỉ khi $a \vdots x$ và $b \vdots x$.

Ví dụ 1. Ư(12) = {1, 2, 3, 4, 6, 12}, Ư(8) = {1, 2, 4, 8}

Thì ước chung của 12 và 8 là 1, 2, 4, kí hiệu ƯC(8,12) = {1, 2, 4}.

Cách tìm ước chung của $a$ và $b$.

  • Tìm tập các số là ước của $a$, tập các ước của $b$.
  • Tìm các phần tử của của hai tập trên ta được tập ước chung của $a$ và $b$.

Ví dụ 2. Tìm ước chung của 24 và 30.

Ta có Ư(24) = {1, 2, 3, 4, 6, 8, 12, 24}, Ư(30) = {1, 2, 3, 5, 6, 15, 30}

Khi đó ƯC(24,30) = {1, 2, 3, 6}.

Ước chung lớn nhất

Ước chung lớn nhất của hai hay nhiều số là số lớn nhất trong tập hợp các ước chung của các số đó.

Kí hiệu ước chung lớn nhất của $a$ và $b$ là ƯCLN(a,b)

Ví dụ 3. ƯC(24,30) = {1, 2, 3, 6}, ƯCLN(24,30) = 6.

Ví dụ 4. Các bạn học sinh lớp 6 A đang lên kế hoạch làm sạch môi trường ở địa phương. Cả lớp có 12 bạn nữ và 18 bạn nam. Các bạn muốn chia lớp thành các nhóm nhỏ gồm cả nam và nữ sao cho số bạn nam và số bạn nữ được chia đều vào các nhóm. Có thể chia được nhiều nhất thành bao nhiêu nhóm học sinh? Khi đó, mỗi nhóm có bao nhiêu bạn nam, bao nhiêu bạn nữ?
Lời giải.

  • Số nhóm được chia phải là ước của cả 12 và 18 .
  • Số nhóm được chia phải là nhiều nhất có thể. Vì vậy, số nhóm được chia là ước chung lớn nhất của 12 và 18 .

Ta có $\mathrm{U}^{\circ} \mathrm{CLN}(12,18)=6$. Do đó cần chia lớp thành 6 nhóm.

Số học sinh trong mỗi nhóm là $(12+18): 6=5$ (học sinh).

Vậy mỗi nhóm có 5 học sinh, gồm 2 nữ và 3 nam.

Cách tìm ước chung lớn nhất của $a, b$ bằng phân tích thành thừa số nguyên tố.

Muốn tìm U’CLN của hai hay nhiều số lớn hơn 1 , ta thực hiện ba bước sau:

  • Bước 1: Phân tích mỗi số ra thừa số nguyên tố.
  • Bước 2: Chọn ra các thừa số nguyên tố chung. Bước 3: Lập tích các thừa số đã chọn, mỗi thừa số lấy với số mũ nhỏ nhất của nó.
    Tích đó là ƯCLN phải tìm.

Ví dụ 5. Tìm ước chung lớn nhất của 24 và 30.

Lời giải.

Ta có $24 = 2^3 \cdot 3$ và $30 = 2 \cdot 3 \cdot 5$.

Ta có ƯCLN (a, b) = 2 \cdot 3 = 6.

Định nghĩa. Hai số có ước chung lớn nhất bằng 1 được gọi là nguyên tố cùng nhau. 

Kí hiệu hai số $a, b$ nguyên tố cùng nhau là (a,b) = 1

Ứng dụng tối giản phân số. Khi rút gọn $\frac{90}{126}$, ta chia cả tử số và mẫu số cho
một ước chung của 90 và 126 để được phân số mới. Tiếp tục
quy trình đó đến khi không rút gọn cho đến khi
tử số và mẫu số của chúng không có ước chung nào khác 1
(tử số và mẫu số là hai số nguyên tố cùng nhau). Khi đó, ta
được một phân số tối giản.

Bài tập rèn luyện

Bài 1. Tìm:
a) $\mathrm{UCLN}(1,16)$;
b) $\operatorname{UCLN}(8,20)$
c) UCLN $(84,156)$;
d) UCLN $(16,40,176)$.
Bài 2. a) Ta có $\mathrm{U}^{\prime} \mathrm{CLN}(18,30)=6$. Hãy viết tập hợp A các ước của 6 . Nêu nhận xét về tập hợp UC $(18,30)$ và tập hợp $\mathrm{A}$.
b) Cho hai số a và b. Để tìm tập hợp $\mathrm{UC}(\mathrm{a}, \mathrm{b})$, ta có thể tìm tập hợp các ước của $\mathrm{U}^{\circ} \mathrm{CLN}(\mathrm{a}, \mathrm{b})$. Hãy tìm UCLN rồi tìm tập hợp các ước chung của:
i. 24 và 30 ;
ii. 42 và 98 ;
iii. 180 và 234 .
Bài 3. Rút gọn các phân số sau: $\frac{28}{42} ; \frac{60}{135} ; \frac{288}{180}$.
Bài 4. Chị Lan có ba đoạn dây ruy băng màu khác nhau với độ dài lần lượt là $140 \mathrm{~cm}, 168 \mathrm{~cm}$ và $210 \mathrm{~cm}$. Chị muốn cắt cả ba đoạn dây đó thành những đoạn ngắn hơn có cùng chiều dài để làm nơ trang trí mà không bị thừa ruy băng. Tính độ dài lớn nhất có thể của mỗi đoạn dây ngắn được cắt ra (độ dài mỗi đoạn dây ngắn là một số tự nhiên với đơn vị là xăng-ti-mét). Khi đó, chị Lan có được bao nhiêu đoạn dây ruy băng ngắn?